Sei sulla pagina 1di 98

MATHEMATICS IN THE MODERN WORLD

Part I. Nature of Mathematics

What does mathematics have to do with nature or art?

A closer look at ancient fabric designs and the woody scales of pine cone also reveals

repeating patterns that can be analyzed in a mathematical way. Mathematics is the

science of patterns and relationships. Mathematics relies on both logic and creativity,

and it is pursued both for a variety of practical purposes and for its intrinsic interest.

“A great discovery solves a great problem


but there is a grain of discovery in the solution of any problem.
Your problem may be modest;
but if it challenges your curiosity
and brings into play your inventive faculties,
and if you solve it by your own means,
you may experience the tension and enjoy the triumph of discovery.”
George Polya

1
MATHEMATICS IN THE MODERN WORLD

2
MATHEMATICS IN THE MODERN WORLD

Chapter

1
MATHEMATICS
IN OUR WORLD
Objectives: At the end of the chapter, the students are expected to:
1. Articulate the importance of mathematics in one‘s life.
2. Express appreciation for mathematics as human endeavors.
3. Argue about the nature of mathematics, what it is, how is it
expressed, represented, and used.
4. Identify patterns in nature and regularities in the world.

Lessons:
1.1 The Meaning of Mathematics
1.2 Mathematics as a Study of Patterns
1.3 Mathematics in Nature

Lesson 1.1 The Meaning of Mathematics

Mathematics may be viewed in different perspectives. In its wildest


significance, it is the development of all types of formal deductive
reasoning. Generally, it is said to be the science of calculation. Others view
it as a science of numbers and space and others say that it is a science of
measurement, quantity and magnitude. Locke said ―Mathematics is a way
to settle in the mind of children a habit of reasoning.” It is a discipline
investigating ―formal structures‖ (Bernays), it is the ―science of orders‖
(Russell), it is the ―science of order in progression‖ (Hamilton). Mathematics

3
MATHEMATICS IN THE MODERN WORLD

has been seen also as a logical construct that is based on a lot of axioms of
either set theory or number theory.

Traditionally, mathematics is presented deductively at school. And is


often perceived as well structured and problems are algorithmically
approached. Merriam dictionary defines mathematics as the science of
numbers and their operations, interrelations, combinations,
generalizations, and abstractions and of space configurations of their
structure, measurement, transformations and generalizations.

Mathematics is derived from the ancient word manthanein meaning


"to learn". The Greek root mathesis means "knowledge" or its other form
máthema meaning science, knowledge, or learning, and mathematikós or
mathemata means "fond of learning". These might have been the notion of
the early mathematicians and philosophers that is why they continue to
seek for knowledge and the truth. Mathematics could then be defined as a
desire for a particular kind of knowing. Knowing that is self-contained on
the individual or may be seen as an autonomous thinking (Schaaf, 1963).

Mathematics is described in so many ways that fits within the area


of human knowledge. Basically it is seen as a study of patterns and
relations. It is also a way of thinking. Mathematics is seen as an art which
is characterized by order and internal consistency. It is a language that
uses carefully defined terms and symbols. Thus, mathematics is a tool
(Reys, Lindquist, Lambdin, Smith and Suydam, 2004).

Mathematics has five basic characteristics‘ namely: precision,


definition, reasoning, coherence, and purposefulness. They are not
independent of each other. It is precise in the sense that mathematical
statements are clear and unambiguous. It is clear what is known and what
is not known. Definitions abound in mathematics. It is the bedrock of
mathematical structure and the platform that supports reasoning.
Reasoning is the lifeblood of mathematics. It is the engine that drives
proving and problem solving. Its absence is the root cause of the learning
by rote approach. Concepts and skills are interwoven in mathematics. And
lastly, mathematics is goal-oriented, and for every concept or skill there is a
purpose for it.

Hardy (1941) states that the beauty of mathematics resides in the


fact that mathematics is all about, not just patterns, but patterns of ideas.
Devlin defines mathematics as the ―science of patterns‖ and then more fully

4
MATHEMATICS IN THE MODERN WORLD

as: ―the science of order, patterns, structure, and logical relationships‖


(Devlin, 2001, p. 73). Mathematics has also often been described as the
language of science. Since the mixture of symbols and words is so
powerfully descriptive and communicative perhaps the definition:
―mathematics is the language of the science of order, patterns, structure
and logical relationships‖ may be considered. Whatever form or way one
defines mathematics so long as it becomes meaningful to the user then it
would be a definition for that user.

Assignment 1.1

Divide the class into 9 groups. Each group draws a number which
are the chapters of Ian Stewart‘s Nature‘s Numbers. The group shows a
power point presentation about their topic or how they interpret the topic.

Activity 1.1b

After all of the groups have presented, each one will answer the
following creatively; either through a poem, a song, an art work or through
whatever skill one wishes to:
1. What new ideas about mathematics did you learn?
2. What is it about that have changed your thoughts about it?
3. What is it most useful about mathematics for humankind?

5
MATHEMATICS IN THE MODERN WORLD

Lesson 1.2 Mathematics as a Study of Patterns and Relationships

Mathematics is the science of patterns and relationships. Patterns


provide a sense of order. It allows one to make an educated guess. A lot of
disciplines are based on making hypothesis and hypotheses are often
based on patterns. Assumptions are also based on patterns, recurring
patterns. Thus, the understanding of patterns aids in the development of
mental skills needed in the transformation of ideas to information then to
knowledge.

As a theoretical discipline, mathematics explores the possible


relationships among abstractions without concern for whether those
abstractions have counterparts in the real world. Investigating the patterns
that one finds in numbers, shapes, and expressions would lead to making
mathematical discoveries. Patterns, relationships, and functions constitute
a unifying theme of mathematics. Patterns are an effective way to
demonstrate the relationship between variables. It provides opportunities to
model and to analyze situations. The opportunities to analyze, extend, and
create a variety of patterns and to use pattern-based thinking to
understand and represent mathematical and other real-world phenomena
are of a great essence in developing mathematical skills. It gives
opportunities for problem solving, making and verifying generalizations,
and building mathematical understanding and confidence.

The abstractions can be anything from strings of numbers or letters


to geometric figures to measurements to sets of equations. Inquiries such
as "Does the interval between prime numbers form a pattern?" ―Is there an
expression for the change in the surface area of any regular solid as its
volume approaches zero?‖; needs investigatory approaches of patterns.

In order to recognize patterns, one needs to have an understanding


of critical thinking and logical reasoning, important skills needed by
everyone. These are major components in problem solving and is involved
in data analysis.

A central line of investigation of patterns is identifying in each


sequence a small set of basic ideas and rules evident in them. One usually
looks at differences between succeeding items. The difference may be
arithmetical in nature but not always. At times one investigates the
movement of shapes, or the relation of the sequences of letters. Logical and

6
MATHEMATICS IN THE MODERN WORLD

abstract reasoning are great tools in investigating these mathematical


patterns.

Patterns essentially provides a means of recognizing broader aspects


that can be shoved down in order to arrive at a specific answer to a
particular problem. The benefit of understanding patterns opens many
doors where there is knowledge that can be applied. The study of patterns
gives an opportunity for one to develop the sense of observation,
hypothesizing, experimenting, discovering and creating. Predicting and
estimating and thought-provoking activities of recognizing patterns
emphasizes the role and purpose of mathematics itself.

Common Patterns

A. LOGIC PATTERNS
Logic patterns are related to geometric patterns and number
patterns. It helps us classify objects or figures. One kind of logic patterns
deal with characteristics of various objects. Another kind deals with orders:
there is a sequence of objects and a pattern in the attributes the objects
possess. They are commonly part of aptitude tests. The development of
numeracy literacy and geometric literacy follows after one develops a sense
of patterns involving logic or reasoning through association or through
relation.

Example 1. In a polygon, a diagonal connects two vertices that are


not already connected by an edge.

How many diagonals can be drawn in a hexagon?

7
MATHEMATICS IN THE MODERN WORLD

Example 2. Look at the following pattern.

How many circles will be in the next figure in the pattern?


A. 40 B. 39 C. 36 D. 27

B. NUMBER PATTERNS

It is a list or set of numbers that follow a certain sequence or


pattern. As an arrangement of number, it illustrates a way that it follows a
particular property or pattern. When numbers are arranged in either
ascending or descending order, and have with it basic operations of
mathematics or a certain series of arithmetical operation like addition or
multiplication repeatedly done. An arithmetic sequence is made by adding
the same value each time. The value added is called the common
difference.

Patterns can be in the form of counting up or down and the missing


number is of the form of completing count up or down. Like 1, 5, 9, 13, 17,
....... whose first term is 1 and jumps 4 every time to get the other terms.

Some other types are created in which every successive term is


multiplied (dividing) by the same value each time. For example, 4, 16, 64,
...... is a number pattern in which 4 is the first number and we can
multiply 4 by 4 to get 16 and multiply 16 by 4 to get 64 and so on. This
number pattern is called a geometric number pattern. 32, 16, 8, 4, 2, .... is
again a geometric number pattern in which the fixed number is ½ which is
the multiplier.

There are other number patterns. There are patterns in which the
numbers are in the increasing form. In this pattern, the amount that is
added in the terms, changes every time in predictable manner. For

8
MATHEMATICS IN THE MODERN WORLD

example, consider 4, 5, 7, 10, 14, 19, ..... In the above pattern, the pattern
starts with 4, add 1 and the amount which we can add increase by 1 every
time. Other types are of the nature of the well-known Fibonacci
sequence 0, 1, 1, 2, 3, 5, 8, 13, 21,..... . The Fibonacci sequence is
a series of numbers where a number is found by adding up the two
numbers before it. Starting with 0 and 1, the sequence goes 0, 1, 1, 2, 3, 5,
8, 13, 21, 34, and so forth.

One of the most interesting Number Patterns is Pascal's


Triangle (named after Blaise Pascal, a famous French Mathematician and
Philosopher). To build the triangle, start with "1" at the top, then continue
placing numbers below it in a triangular pattern. Each number is the
numbers directly above it added together.

To recognize any sequence, we have to know how the terms of a


sequence are related. It may be that they are in arithmetic form or growing
pattern or geometric pattern. At times one need to recognize a property or
characteristic the number has.

Let us consider 1, 8, 27, 64,.... Here, we can find the next number as
follows: 8 is , 27 is and 64 is . Thus, we write the sequence as 13, 23,
3 , 4 ,... So, the next terms are 53 = 125 and 63 = 216.
3 3

Example 1: Find the next number in the pattern 45, 48, 51, 54, 57, ___

Solution:
Given pattern is 45, 48, 51, 54, 57, ___

9
MATHEMATICS IN THE MODERN WORLD

Here, the pattern is a repeated addition of 3, we get: 45, 48, 51, 54,
57, 60

In the problem, sixth term is 60 in this pattern which is the missing


term.

Example 2: Find out the missing number in the pattern 67, 74, 81, 88, __,
102

Solution:
Given pattern is 67, 74, 81, 88, __, 102
Here, the pattern is repeated addition of 7, we get 67, 74, 81, 88, __,
102

In the problem, fifth term is 95.


Therefore, the missing term in the above pattern is 95 for the
repeated addition of 7.

Example 3: What is the next number in the pattern 108, 102, 96, ___

Solution:
Given pattern is 108, 102, 96, ___
Here, the pattern is the subtraction of 6, we get 108, 102, 96, 90

In the problem, fourth term is missing and the next term of the given
pattern is 90.
Therefore, the next number in the above pattern is 90 for the
subtraction of 6.

C. GEOMETRIC PATTERN

A geometric pattern is a kind of pattern formed of sequences of


lines and curves to form geometric shapes and figures. A motif, pattern, or
design depicting abstract, nonrepresentational shapes such as lines,
circles, ellipses, triangles, rectangles, and polygons. We generally associate
geometric patterns with wall paper designs and tiling. They could also be
associated to number patterns and predict the next geometric shape that
would follow the sequence.

10
MATHEMATICS IN THE MODERN WORLD

Example 1: Determine what shape, figure, or series of figure would


follow the sequence.

Example 2: Draw the missing figure in the sequence.

11
MATHEMATICS IN THE MODERN WORLD

12
MATHEMATICS IN THE MODERN WORLD

NAME: _____________________________________________ SCORE: _______________


SCHEDULE: _________________________________________ DATE: ________________

Exercise 1.2A

Series of Numbers.

1. Find the missing term in the following sequence: 8, ___, 16, ___, 24, 28, 32.
2. What is the value of n in the following number sequence? 16, 21, n, 31, 36.
3. Find the next term in the sequence: 7, 15, 23, 31, ___.
4. Find the next term in the sequence: 31, 24, 17, 10, ___.
5. Find the next three terms in: -14, -10, -6, -2, ___, ___, ___.
6. Find the 35th term in the arithmetic sequence: 3, 9, 15, 21, …
7. Find the next two terms in the sequence: 5, 2, 8, 3, 11, 4, 14, 5, 17, 6, ___,
___
8. Find the next number: 2, 3, 4, 6, 6, 9, 8, ___
9. What should come in place of the blank space in the following series?
3, 5, 9, 15, 23, 33, ___, 59.
10. What should come in place of the blank space in the following series?
16, 22, 28, 34, ___, 46.
11. What should come in place of the blank space in the following series?
2, 3, 6, 11, 18, 27, ___.
12. What should be come in place of x in the following series?
2, 2, 4, 12, 48, 240, x.
13. Identify the next number in the following series:
2, 8, 26, 62, 122, 212, ___
14. What is the next number in the pattern?
76, 78, 80, 82, 84, 86, ___
15. What is the next number in the pattern?
4, 12, 36, 108, ___

13
MATHEMATICS IN THE MODERN WORLD

14
MATHEMATICS IN THE MODERN WORLD

NAME: _____________________________________________ SCORE: _______________


SCHEDULE: _________________________________________ DATE: ________________

Exercise 1.2B

I. Identify the next shape in the series.

1.

a. b. c. d.

2.

a. b. c. d.

3.

a. b. c. d.

15
MATHEMATICS IN THE MODERN WORLD

II. Which of the figures can be used to continue the series given below?
1.

a. b. c. d. e.

2.

a. b. c. e.

III. Which of the figures, do you think best fits the series below?

a. b. c. d.

16
MATHEMATICS IN THE MODERN WORLD

Lesson 1.3 Mathematics in Nature

Euclid said that "The laws of nature are but the mathematical
thoughts of God." Galileo affirmed by stating that ―Mathematics is the
language in which God has written the Universe.‖

Mathematics is everywhere, it is seen anywhere in the universe. With


the development of a formal system of thought for recognizing, classifying,
and exploiting patterns; one could systematize and organize these ideas of
patterns. It would be here that we could discover great secrets of nature‘s
patterns. They are not just there to be admired, they are vital clues to the
rules that governs the natural process.

Analyzing thoroughly, having the essentials of math as our basis, we


could further discover mathematics in our world and unravel the mystery
of the universe. The majority of our knowledge of mathematics and modern
science is strictly based and supported on our observations of our
environment. What was once seen as the randomness of nature is now
distinguished as the intricate applications of mathematics and illustrates
the complexities of our natural world. Here are a very few properties of
mathematics that are depicted in nature.

A. SHAPES
Geometry is the branch of mathematics that basically
describes shapes and establishes the relationships between them. Figures
with regular shapes are categorized as polygons. Polygons are fascinating,
especially when they are approximated in nature. When looking carefully,
one can see them all around us.

Spatial patterns can be represented by a fairly small collection of


fundamental geometrical shapes and relationships that have corresponding
symbolic representation. To make sense of the world, the human mind
relies heavily on its perception of shapes and patterns. The artifacts around
us (such as buildings, vehicles, toys, utensils and basic things we use in
life) and the familiar forms we see in nature (such as animals, leaves,
stones, flowers, and the moon and sun) can often be characterized in terms
of geometric form. Some of the ideas and terms of geometry have become
part of everyday language. Although real objects never perfectly match a
geometric figure, they more or less approximate them. The properties and
characteristics about geometric figures and relationships can be associated
to objects. For many purposes, it is sufficient to be familiar with points,

17
MATHEMATICS IN THE MODERN WORLD

lines, planes; triangles, rectangles, squares, circles, and ellipses;


rectangular solids and spheres; relationships of similarity and congruence;
relationships of convex, concave, intersecting, and tangent; angles between
lines or planes; parallel and perpendicular relationships between lines and
planes; forms of symmetry such as displacement, reflection, and rotation;
and the Pythagorean theorem.

Both shape and measurement (magnitude) or scale can have


important consequences for the performance of systems. For example,
triangular connections maximize rigidity, smooth surfaces minimize
turbulence, and a spherical container minimizes surface area for any given
mass or volume. Changing the size of objects while keeping the same shape
can have profound effects owing to the geometry of scaling: Area varies as
the square of linear dimensions, and volume varies as the cube. On the
other hand, some particularly interesting kinds of patterns known as
fractals look very similar to one another when observed at any scale
whatever—and some natural phenomena (such as the shapes of clouds,
mountains, and coastlines) seem to be like that.

Some Common Shapes Visible in Nature:

Sphere. A sphere is a perfectly round geometrical object


in three-dimensional space, such as the shape of a
round ball. The shape of the Earth is very close to that
of an oblate spheroid, a sphere flattened along the axis
from pole to pole such that there is a bulge around the
equator.

Hexagons. Hexagon is a six-sided closed-polygons, 2-


dimensional. For a beehive, close packing is important
to maximize the use of space. Hexagons fit most closely
together without any gaps; so hexagonal wax cells are
what bees create to store their eggs and larvae.

Cones. A cone is a three-dimensional geometric shape


that tapers smoothly from a flat, usually circular base to
a point called the apex or vertex. Volcanoes form cones,
the steepness and height of which depends on the
runniness (viscosity) of the lava. Fast, runny lava forms
flatter cones; thick, viscous lava forms steep-sided
cones.

18
MATHEMATICS IN THE MODERN WORLD

Parallel lines. In mathematics, parallel lines stretch to


infinity, neither converging nor diverging. The parallel
dunes in the Australian desert aren't perfect - the
physical world rarely is.

B. SYMMETRY
Symmetry is a type of invariance: a property that something does not
change under a set of transformations. It is a mapping of the object onto
itself which preserves the structure. Symmetry in everyday language refers
to a sense of harmonious and beautiful proportion and balance. Although
these two meanings of "symmetry" can sometimes be told apart, they are
related. Plainly, symmetry is when a figure has two sides that are mirror
images of one another. It would then be possible to draw a line through a
picture of the object and along either side the image would look exactly the
same. This line would be called a line of symmetry.

There are Two Kinds of Symmetry.

One is bilateral symmetry in which an object has two sides that are
mirror images of each other. The human body would be an excellent
example of a living being that has bilateral symmetry.
The other kind of symmetry is radial symmetry. This is where there
is a center point and numerous lines of symmetry could be drawn. The
most obvious geometric example would be a circle.

C. PATTERNS
Though every living and non-living thing of the world may seem to
follow a pattern of its own, looking deeply into the geometry and
mechanism of the pattern formation can lead you to broadly classify them
into merely two categories:
 Self-organized patterns/ Inherent organization
 Invoked organization

Self-Organized patterns
A self-organizing pattern, follows a simple set of rules, and they use
only local information to determine how a particular subunit evolves. They
are represented by successive patterns. This pattern can be represented as
successive horizontal rows; the 'successor' pattern is just under its
predecessor. When the basic rule just defined is applied to that row (the

19
MATHEMATICS IN THE MODERN WORLD

active row) and then to subsequent rows, a complex pattern develops.


Thus, self-organization is a process in which pattern at the global level of a
system emerges solely from numerous interactions among the lower level
components of the system. Moreover, the rules specifying interactions
among the system‘s components are executed using only local information,
without reference to the global pattern. In other words, the pattern is an
emergent property of the system, rather than a property imposed on the
system by an external influence.

Therefore, if rules are to be useful for understanding the patterns in


life, such as the stripes on a zebra's coat, there must be a specific rule. The
zebra's coat alternates in contrasting areas of light and dark pigmentation.
Hence the patterns of the zebra's coat reflect the early interaction of
chemicals as they diffused through the embryonic skin.

Invoked Organization
Not all patterns that occur in nature arise through self-organization.
A weaver bird uses its own body as a template as it builds the
hemispherical egg chamber of the nest. A spider when creating a web
follows a genetically determined recipe in relation to its sticky orb and the
various radii and spirals it creates. A similar invoked organization is that of
the honeycomb made by bees. In these cases, the structures are built
something of an architect that oversees and imposes order and pattern.
There are no sub units that interact with one another to generate a pattern.
Each of the animals acts like a stonemason or laborer, measuring, fitting,
and moving pieces into place.

20
MATHEMATICS IN THE MODERN WORLD

This is an example of a social insect


architecture. The wasp nests, an example of
invoked organization in nature.
More than this biological system existing
on the patterns created in nature, there is a
great mathematics embedded in it aside from the
plain geometric figure. The geometry of the
patterns could also be linked to mathematical
numbers directly or indirectly. The series of
numbers seem to have been forced to them. The
Fibonacci numbers or sequence and the Golden
ratio are but the basic example of them.
Leonardo Fibonacci began the study of this sequence by posing the
following problem in his book, Liber Abaci: “How many pairs of rabbits
will be produced in a year, beginning with a single pair, if in every
month each pair bears a new pair which becomes productive from the
second month on?”
This problem gives rise to the sequence 1, 1, 2, 3, 5, 8, 13, ... in
which any term after the first two can be found by summing the two
previous terms. In functional notation we could write f(n) = f (n - 1) + f (n -
2) using f(0) = 1 and f(1) = 1. Interestingly, the ratio between two
consecutive terms of this series tends to the number 1.61803399. It is a
number commonly encountered when taking ratios of distances in simple
geometric figures such as pentagons, decagons and dodecagons. It is
denoted by PHI, and is called the divine proportion, golden mean, or golden
section.
Surprisingly, the Fibonacci numbers are found to occur in a wide
variety of unexpected situations. Indeed, their occurrence is very common
in nature including images of flowers and of fruits and vegetables. There
are many examples of biological growth involving the Fibonacci numbers
such as branch in trees, the reproduction of bees, the pattern of petals in
many flowers and plants. They also form the number of leaves and seed
grains of many plants. A unique and beautiful spiral pattern is observed in
mature sunflowers which also display Fibonacci numbers. In particular, it
is seen that the sunflower has F(10) = 55 spirals in one direction and F(11)
= 89 spirals in the other direction representing fairly large Fibonacci
numbers. The scale patterns on pineapples and pine cones provide
excellent examples of Fibonacci numbers. The scales of pineapples are,
indeed, hexagonal in shape.

21
MATHEMATICS IN THE MODERN WORLD

Another geometrical figure that is commonly associated with Phi is


the Golden Rectangle. This particular rectangle has sides A and B that are
in proportion to the Golden Ratio. It has been said that the Golden
Rectangle is the most pleasing rectangle to the eye. If we take the isosceles
triangle that has the two base angles of 72 degrees and we bisect one of the
base angles, we should see that we get another Golden triangle that is
similar to the first. If we continue in this fashion, we should get a set of
Whirling Triangles.

Out of these Whirling Triangles, we are


able to draw a logarithmic spiral that will
converge at the intersection of the two blue
lines, (see figure).

A logarithmic spiral that is a commonly


observed pattern in nature.

A pine cone (leftmost figure) exhibits the


pattern of spirals of both directions – 13 clockwise and 8 counterclockwise
(13 and 8 are consecutive terms of the Fibonacci Series). The seed of the
cone flower (middle figure) follows a logarithmic spiral pattern. The shell of
a snail (rightmost figure) is also in the shape of spiral.

The world around us seems to make up of several distinct patterns,


evolving various complex steps of formation. However, looking more deeply
we see many similarities and resemblances. The numerous models
explained above have no experimental proof and may not be correct, but
they definitely show linkages between patterns formed under highly
contrasting natural conditions e.g. (a zebra coat and sand dunes) and also
show that the mechanisms between the formations of these patterns need
not necessarily be complex

22
MATHEMATICS IN THE MODERN WORLD

The Golden Ratio

Mathematics deals with harmony. Harmony is the balance and order


within a system. It is beauty, a quality that is present in a thing or person
that gives intense pleasure or deep satisfaction to the mind. The pleasure
maybe arising from sensory manifestations (as shape, colour, sound, etc.),
a meaningful design or pattern, or something else (as a personality in
which high spiritual qualities are manifest). The attraction that one has is
dependent on the ratio one sees. The ratio is of high degree of proportional.
Leonardo da Vinci's drawings of the human body emphasized its
proportion. The ratio of the following distances is of the Golden Ratio: (foot
to navel):(navel to head).
The word golden section was known to the Greek mathematicians as
‗division of a line in mean and extreme ratio‘ that appeared in the
celebrated quotation of Johann Kepler (1571–1630): ―Geometry has two
great treasures: one is the theorem of Pythagoras; the other, the division of
a line in extreme and mean ratio. The first we may name as a measure of
gold, the second we may name as a precious jewel.‖

(a) (b)
A x C y B A

x z
x 0

F x E y D B x D y C
The golden ratio (or the golden number or the golden section) is
defined by dividing a line segment AB = a by the point C, as shown in
Figure 1, into two unequal parts x and y such that (y < x) in such a way
that the ratio of the larger part x to the smaller part y is equal to that of the
total length x + y = a to the larger segment x,
that is,

23
MATHEMATICS IN THE MODERN WORLD

The Golden Ratio is a product of early Greek mathematics, which


saw the development of the related concepts of ratio and proportion. If p
and q are any two natural numbers or positive integers, represents the
ratio of p to q. A ratio is the quotient of two numbers or quantities. When
two ratios are equated, one obtains a proportion. Thus, represents a
proportion. Proportions of the type are known as continuous
proportions. One particular type of continuous proportion, ( )
, was
very special to the ancient Greeks, who named it the Golden Proportion.
The Golden Proportion is the simplest of continuous proportions because it
has only two unknowns and uses the most basic arithmetic operation,
addition.
It follows from the Golden Proportion that:
( )

Simplifying the right fraction:

Multiplying both sides of the equation by :

( )

The ratio derived from the Golden Proportion was called the
( √ )
Golden Ratio by the ancient Greeks, and its exact value is obtained
by solving for x, the Golden Ratio being represented by x.
The Golden Ratio became the standard of perfection in Greek art and
architecture. This tradition was continued by the Romans and carried on
into the Middle Ages, where the Golden Ratio is represented in the
architecture of the great cathedrals. The great artists and architects of the
Renaissance inherited this cultural legacy and passed it on to their heirs in
the Modern Age.
The description of this proportion as Golden and Divine is fitting
perhaps because it is seen by many to open the door to a deeper
understanding of beauty and spirituality in life. That's an incredible role
for a single number to play, but then again this one number has played an
incredible role in human history and in the universe at large.
It is a mathematical fact that the ratio of a Fibonacci number to the
number that precedes it in the sequence approaches the Golden Ratio at
the limit of the sequence. It is also a mathematical fact that any Fibonacci

24
MATHEMATICS IN THE MODERN WORLD

number can be represented by a general formula, known as Binet's


Formula that incorporates the Golden Ratio. Letting F(n) represent the nth
number of the Fibonacci sequence,
√ √
( ) (( ) ( ) )

The Fibonacci Sequence can be derived from Pascal's Triangle by
adding numbers in Pascal's Triangle diagonally. The diagonal sequence of
numbers on the upper right represents the first nine terms of the Fibonacci
sequence (excluding the initial zero). They are seen to be the sums of
numbers in Pascal's Triangle connected by diagonal lines.

25
MATHEMATICS IN THE MODERN WORLD

26
MATHEMATICS IN THE MODERN WORLD

Chapter

MATHEMATICAL

2 LANGUAGE AND
SYMBOLS
Objectives: At the end of the chapter, the students are expected to:
1. Discuss the language, symbols and conventions of mathematics.
2. Explain the nature of mathematics as a language.
3. Perform operations on mathematical expressions correctly.
4. Acknowledge that mathematics is a useful language.
5. Use different types of reasoning to justify statements and
arguments made about mathematics and mathematical concepts.
6. Write clear and logical proofs

Lessons:
2.1 The Fundamental Elements of the Language of Mathematics
A. Numbers
B. Sets
C. Relations
D. Functions
E. Operations
2.2 Mathematics as a Language

27
MATHEMATICS IN THE MODERN WORLD

Language as defined by dictionaries is a body of words or symbols


and the systems for their use common to a people who are of the same
community or nation, the same geographical area, or the same cultural
tradition. It is a systematic means of communicating by the use of sounds
or conventional symbols (Chen, 2010, p.353). Aside from the symbols, one‘s
understanding of the language is enormously enhanced by a knowledge of
basic grammar or syntax. This understanding is essential for anybody who
wants to learn more about the language and its usage. It would be then
that one is able to communicate with others. The same is true of
mathematics. One needs to learn and understand the mathematical
language in order to think, to comprehend, and to communicate
mathematically. Mathematics as a language is composed of vocabularies
consisting of symbols and words and grammar which is a set of rules on
how to use these symbols.

Mathematical symbols combined with words can convey complex,


powerful ideas more efficiently than any other language. It has been a tool
and the great instrument for knowledge and wisdom to develop and
flourish. The history of man‘s civilization is in parallel with the history and
development of mathematics. It has become a universal language.
Thus, the object of this section is to explain the most important
mathematical ―parts of speech‖, some of which are similar to those of
natural languages and others quite different.

Lesson 2.1. The Fundamental Elements of the Language of

Mathematics
Mathematics deals with ideas -- relationships, quantities, processes,
ways of figuring out certain kinds of things, reasoning, generalizing and
many more. It uses words, but it is not about words. The ideas when they
are shared need to have means for they be conveyed. A plain knowledge of
the mathematical term does not imply that we know the language. Knowing
―denominator‖ and ―addend‖ is not math and does not make one
mathematical. Words help us communicate.

The nature of the language of mathematics has a number of


distinctive features that is different from the common spoken language. It
includes aspects which are particular to mathematics and as applied to
other disciplines.

28
MATHEMATICS IN THE MODERN WORLD

The language of mathematics has a specialist mathematical


vocabulary, which includes:
 Technical terms specific to mathematics (e.g. equilateral, quotient,
probability);
 Specialist use of more general terms (e.g. line, factor, frequency);
 Mathematical terms that we use every day for conveying ideas
(e.g. function, expression, difference, area).

It also includes a special syntax, particularly in relation to the


expression of logical relationships. Thus, the use of and, of, or, a, if and
then to define mathematical relationships are all significant. It involves the
use of mathematical symbols. Such symbols range from numerals to more
specialized notation. These symbols have a syntax of their own.
Mathematics has a different way of communicating, including written and
spoken forms of mathematical explanation, proof or definition, as well as
text types like word problems. These broader ways of using language are
important in expressing mathematical ideas and reasoning.

We are already familiar with most of the basic symbols that are
used. For example, the digits 0, 1, 2, 3, 4, 5, 6, 7, 8, 9; have become part of
our everyday lives. With it is the extension of our notion about numbers,
we understand its meaning.

These symbols in mathematics are precise form of shorthand


notation. We need to be confident when using these symbols, and to gain
that confidence we need to understand their meaning. To understand their
meaning there are two things to help us:
 Context - this is the context in which we are working, or the
particular topics being studied, and
 Convention - where mathematicians and scientists have decided
that particular symbols will have particular meaning.

The advantage of mathematical notation, both symbolic and


graphical, is that it is highly compact (conveying a lot of information and
ideas in very little space) and focused (conveying the important information
for the current situation and omitting the rest).

A. NUMBERS
A number is a symbol, either a figure or word, that expresses a
certain value or a specified quantity that is determined by count. There are
several kinds of numbers that in combination with a logic for interrelating

29
MATHEMATICS IN THE MODERN WORLD

them form interesting abstract systems and can be useful in a variety of


very different ways. The age-old concept of number probably originated in
the need to count how many things there were in a collection of things.

The Hindu-Arabic number system, as commonly used today, is


based on ten symbols (0, 1, 2, . . . 9) and rules for combining them in
which position is crucial. Its position determines its place value. The basic
unit are ones, tens and hundreds. They may either be in the unit or
thousands, millions or more.

There are different kinds of numbers. The numbers that come from
counting things are whole numbers, which are the numbers we mostly use
in everyday life. A whole number by itself is an abstraction for how many
things there are in a set but not for the things themselves. "Three" can refer
to chairs, trees, people, or anything else. But in most practical situations,
we want to know what the objects are, as well as how many there are.
Thus, the answer to most calculations is a magnitude—a number
connected to a label. If some people traveled 165 miles in 3 hours, their
average speed was 55 miles per hour, not 55. In this instance, 165, 3, and
55 are numbers; 165 miles, 3 hours, and 55 miles per hour are
magnitudes. The labels are important in keeping track of the meanings of
the numbers.

Fractions are numbers we use to stand for a part of something or a


comparison of two quantities. One common kind of comparison occurs
when some magnitude such as length or weight is measured—that is, is
compared to a standard unit such as a meter or a pound. Fractions may be
written in decimal form, specifically if it is a terminating decimal. For
example, the ordinary fraction 3/4 and the decimal fraction 0.75 both
represent the same number. However, the two expressions may have
somewhat different implications: 3/4 could be used to simply mean closer
to 3/4 than to 2/4 or 4/4, whereas 0.75 may imply being closer to 0.75
than to 0.74 or 0.76—a much more precise specification.

More flexibility in mathematics is provided by the use of negative


numbers, which can be thought of in terms of a number line. A number
line lays consecutive numbers at equal intervals along a straight line
centered on zero. The numbers on one side of zero are called positive, and
those on the other side, negative. If the numbers to the right of zero are
positive, the numbers to the left of zero are negative; if distance above sea
level is positive, distance below sea level is negative; if income is positive,

30
MATHEMATICS IN THE MODERN WORLD

debt is negative. If 2:15 is the scheduled time of lift-off, 2:10 is "minus 5


minutes." The complete range of numbers—positive, zero, and negative—
allows any number to be subtracted from any other and still give an
answer.

Numbers have many different uses, some of which are not


quantitative or strictly logical. In counting, for example, zero has a special
meaning of nothing. Yet, on the common temperature scale, zero is only an
arbitrary position and does not mean an absence of temperature (or of
anything else). Numbers can be used to put things in an order and to
indicate only which is higher or lower than others—not to specify by how
much (for example, the order of winners in a race, street addresses, or
scores on psychological tests for which numerical differences have no
uniform meaning). And numbers are commonly used simply to identify
things without any meaningful order, as in telephone numbers and as used
on athletic shirts and license plates.

B. SETS
In an attempt to better understand the universe, ancient
astronomers classified certain groups of stars as constellations. In regional
sports competitions, groupings are done to facilitate the management of
activities. Schools are organized into districts or areas, too. It is still
extremely helpful to classify items into groups that enable us to find order
and meaning in our complicated world.

Basic Properties of Sets:

Definition: A set is a well-defined collection of distinct objects. It is


usually represented by capital letters. A set is said to be well-defined
if the elements in a set are specifically listed or if its elements are
described to determine whether an object in question is an element
or not an element of the set. The objects that belong in a set are the
elements, or members, of the set. The objects of a set are also
separated by commas.
Examples:
A = {a, e, i, o, u}
B = {set of plane figures}
C = {Ca, Au, Ag}

31
MATHEMATICS IN THE MODERN WORLD

A set can be represented by listing its elements between braces:


A = {1,2,3,4,5,6,7,8,9,0}. This is the tabular or roster form. The
symbol ∈ is used to express that an element is part of a set (or
belongs to a set), for instance 3 ∈ A. An alternative way to define a
set, called set builder notation, is by stating a property (predicate)
P(x) verified by exactly its elements, for instance A = {x ∈ Z | 1 ≤ x ≤
5} = ―set of integers x such that 1 ≤ x ≤ 5‖—i.e.: A = {1,2,3,4,5}. In
general: A = {x ∈ U | p(x)}, where U is the domain of discourse in
which the predicate P(x) must be interpreted, or A = {x | P(x)} if the
domain of discourse for P(x) is implicitly understood. {x|…}. is read
as ―x such that. In set theory the term universal set is often used in
place of ―domain of discourse‖ for a given predicate. Others call it as
the rule form.

Illustrations:

Roster Rule
{1,2,3} {x | x is a natural number less than 4}
{ } * + { | +
{2,4,6,8,10} {x | x is an even integer between 0 and 12}
{0,3,6,9} {k | k is a multiple of 3 between -1 and 12}

Some important sets are the following:


1. N = {1,2,3,···} = the set of natural numbers.
2. W = {0,1,2,3,···} = the set of whole numbers.
3. Z = {−3,−2,−1,0,1,2,3,···} = the set of integers.
4. Q = the set of rational numbers (terminating or repeating
decimals).
5. Q‘ = the set of irrational numbers (nonterminating, nonrepeating
decimals).
6. R = the set of real numbers.
7. C = the set of complex numbers.

Definitions Regarding Sets

A set is finite if the number of elements in the set is a whole


number. The cardinal number of a finite set is the number of elements in
the set. It contains only a countable number of elements.
Example:
A = {barangays in Baguio City}

32
MATHEMATICS IN THE MODERN WORLD

A set is infinite if the counting of elements has not end. The set of
integers Z or positive integers N (or natural numbers), negative integers Z-,
and nonnegative integers (or whole numbers) are infinite sets.
Z = {…, -2, -1, 0, 1, 2, ….}; Z- = {-1, -2, -3…}; N = {1, 2, 3…}; W = {0, 1,
2,…}

The set of all elements that are being considered is called the
universal set. We will use the letter U to denote the universal set. If A = {1,
2, 3} and B = {3, 4, 5} then the universal set U, considering no other set
present is U = {1, 2, 3, 4, 5}.

The empty set, or null set, is the set that contains no elements. The
symbol Ø or { } is used to represent the empty set. As an example of the
empty set, consider the set of natural numbers that are negative integers.
The set {Ø} is not empty since it contains one element.

Set A, A= { Ø }, is not an empty set since it has one element. The set
with only one element is a unit set.

Set A is equal to set B, denoted by A = B, if and only if A and B have


exactly the same elements.
Example:
A  h, o, p, e
B  p, o, e, h

Set A is equivalent to set B, denoted by A ~ B, if and only if A and B


have the same number of elements. The cardinality of the two sets is the
same.
Example:
A  l , o, v, e
B   ,  ,  , 
The sets C = {a, b, c} and D = {4, 5, 6} are equivalent sets. Also, {2} ~
{Ø}.
Equal sets are equivalent, but not vice versa.

Sets that have common elements are joint sets. The sets A = {4, 5,
6} and B = {6, 10, 11} are joint sets, since 6 is a common to both A and B.
The sets C = {r, I, c, h} and D = {p, o, b, r, e} are joint sets because r is
common to both C and D.

33
MATHEMATICS IN THE MODERN WORLD

Two sets are disjoint if they have no common elements. The set E =
{a , b, c} and F = {e, f, g} are disjoint sets, since no element is common. The
set {0} and {Ø} are also disjoint sets. The positive odd integer ZO = {1, 3, 5,
…} and the nonnegative even integers Ze = {0, 2, 4, …} are disjoint sets. Also,
the negative integers Z- = {-1, -2, …} and the nonnegative integers W = {0, 1,
2, …} are disjoint sets.

Set A is a subset of set B denoted by A ⊂ B, if every element of A


belongs to B.

In Symbol, A ⊂ B if x € A, then x € B.

Aside from the definition, if there is at least one element found in B


but not in A, then A is a proper subset of B denoted by A ⊂ B. There are
two improper subsets of any given set, the empty set and the set itself. The
power set P of A, denoted by P(A) is defined as the set of all subsets of A.

The following generalizations are consequences of the definition.


a. Every set is a subset of itself, i.e. A ⊂ A.
b. An empty set is always a subset of every set, i.e. Ø ⊂ A.
c. The sets {Ø} and {0} are not empty, since each contains one
element.

Take note that the number of subsets of a given set is 2n where n


denotes number of elements of the set. In the preceding example, A has 22
= 4 subsets.
Example:
The set B = {a, b, c} has 23 subsets. The power set has the
element with breakdown as follows:
Improper subset with 3 elements: {a, b, c}, or B
Proper subsets with 2 elements: {a, b}, {a, c}, {b, c}
Proper subsets with 1 element: {a}, {b}, {c}
Improper subset with no element: { }
Power set of B = P (B) = {B, Ø, {a, b}, {a, c}, {b, c}, {a}, {b}, {c}}

The complement of a set A, denoted by A‘, is the set of all elements


of the universal set U that are not elements of A.

There are two fundamental results concerning the universal set and
the empty set. Because the universal set contains all elements under
consideration, the complement of the universal set is the empty set.

34
MATHEMATICS IN THE MODERN WORLD

Conversely, the complement of the empty set is the universal set, because
the empty set has no elements and the universal set contains all the
elements under consideration. Using mathematical notation, we state these
fundamental results as follows:
U’ = Ø and Ø‘ = U.

The English logician John Venn (1834–1923) developed diagrams


that can be used to illustrate sets and relationships between sets. This
diagram facilitated one‘s conceptualization of the sets and relations within
it. It is called the Venn Diagram. In a Venn diagram, the universal set is
represented by a rectangular region and subsets of the universal set are
generally represented by oval or circular regions drawn inside the
rectangle. Others would prefer different types of polygons to emphasize
differences between them. The Venn diagram below shows a universal set
and one of its subsets, labeled as set A. The size of the circle is not a
concern. The region outside of the circle, but inside of the rectangle,
represents the set A‘.

U
A

A Venn Diagram

The operations on sets behave in a manner somewhat similar to


the basic operations on numbers.

Four Basic Operations on Sets:


1. Union of Sets A and B

A ∪ B = sets of all elements found in A or B or both = {x | x ∈ A or x


∈ B}
Example: {a, b, c, d, e} ∪ {b, e, f, g} = {a, b, c, d, e, f, g}
In general: A ∪ U = U, A ∪ Ø = A, A ∪ A = A

35
MATHEMATICS IN THE MODERN WORLD

2. Intersection of Sets A and B


A ∩ B = set of all elements common to both A and B = {x | x ∈ A and
x ∈ B}
Example: {1, 2, 3, 4} ∩ {0, 2, 3, 4, 9} = {2, 3, 4}
In general: A ∩ U = A, A ∩ Ø = Ø, A ∩ A = A

3. Complement of A
A‘ = set of all elements found in the universal set but not in A = { x |
x ∈ U and x not ∈}
Example: A = {1, 2}, U = {1, 2, 3, 4, 5} A‘ = {3, 4, 5}
In general: A‘ ∪ U = U, A‘ ∪ A = U

4. Difference of sets A and B


A – B = set of all elements found in A but not in B = {x | x ∈ U and x
∈ B}
B – A = {x | x ∈ B and x ∈ A}
Example 1:
A = {4, 5, 6, 7}, B = {1, 6, 7, 8, 9}
A – B = {4, 5}
B – A = {1, 8, 9}
Example 2:
C = {b, r, a, I, n}; D = {r, a, I, n, d, r, o, p, s}
C – D = {b}
D – C = {d, r, o, p, s}
In general: A – B = B‘ ∩ A; B – A = A‘ ∩ B

Laws of Sets
Sets involving the operations union, intersection, complement and
difference satisfy properties which we shall refer to as the laws of sets.
1. Commutative Law – The order in which the sets are taken does not
affect the result.
A∪B=B∪A A∩B=B∩A
Examples:
{2} ∪ {3} = {3} ∪ {2}; {2} ∩ {3} = {3} ∩ {2}

2. Associative Laws – The grouping in which the sets are taken does
not affect the result.
A ∪ (B ∪ C) = (A ∪ B) ∪ C A ∩ (B ∩ C) = (A ∩ B) ∩ C
Examples:
[{a} ∪ {b, c}] ∪ {c, e, f} = {a} ∪ [{b, c} ∪ {c, e, f}]

36
MATHEMATICS IN THE MODERN WORLD

3. Identity Laws – A set operated to another set called the identity


element gives the set itself.
A ∪ Ø = A, for union of sets, the identity is the empty set.
A ∩ U = A, for intersection of sets, the identity element is the
universal set.

4. Inverse or Complement Laws – This involves inside and outside of


a set.
A ∪ A‘ = U A ∩ A‘ = Ø

5. Distributive Laws – These laws involve three sets with two different
operations, distributing the first operation over the second one.

A ∪ (B ∩ C) = (A ∪ B) ∩ (A ∪ C); Left Distributive Law of ∪ over ∩.


A ∩ (B ∪ C) = (A ∩ B) ∪ (A ∩ C); Left Distributive Law of ∩ over ∪.
(A ∩ B) ∪ C = (A ∪ C) ∩ (B ∪ C); Right Distributive Law of ∪ over ∩.
(A ∪ B) ∩ C = (A ∩ C) ∪ (B ∩ C); Right Distributive Law of ∩ over ∪.

37
MATHEMATICS IN THE MODERN WORLD

38
MATHEMATICS IN THE MODERN WORLD

NAME: _____________________________________________ SCORE: _______________


SCHEDULE: _________________________________________ DATE: ________________

Exercise 2.1.A

A. Write if the set is empty, unit, finite or infinite.


1. A = {Tourist spots in the Philippines} 1. ________________
2. B = {Students in your class who are 9 years old} 2. ________________
3. C = {Midpoint of segment AB} 3. ________________
4. D = {TV Networks in the Philippines} 4. ________________
5. E = {Whole Numbers Greater than 3 and less than 4} 5. ________________
6. F = {Raindrops} 6. ________________
7. G = {Your Father} 7. ________________
8. H = {Set of Whole Numbers Between -1 and 0} 8. ________________
9. I = {Complex Numbers} 9. ________________
10. J = {Steps in Solving any Problem} 10. ________________

B. Determine if the pair of sets are equal, equivalent, joint or disjoint.


A  letters in the word loop
1. 1. ________________
B  letters in the word pool
C  vowels in the English alphabet 
2. 2. ________________
D  letters in the word seven
E  letters of the word faith 
3. 3. ________________
F  letters of the word hope
G  whole numbers less than 10
4. 4. _______________
H  whole numbers greater than 10 but less than 15
I  the first 8 counting numbers
5. 5. ________________
J  whole numbers between 0 and 9
K  u, r , b, a, n
6. 6. ________________
L  r , u, r , a, l
M  u, r , b, c
7. 7. ________________
N  i, n, v, u
O  letters of the word kind
8. 8. ________________
P  letters of the word kindle
Q  vowels in the Filipino alphabet 
9. 9. ________________
R  vowels in the word miscellane ous
S  set of odd numbers between 0 to 10
10. 10. ________________
T  set of even numbers between 0 to 10

39
MATHEMATICS IN THE MODERN WORLD

40
MATHEMATICS IN THE MODERN WORLD

NAME: _____________________________________________ SCORE: _______________


SCHEDULE: _________________________________________ DATE: ________________

Exercise 2.1.B

U  1,2,3,4,5,6,7,8,9,10
A  1,2,3,4,5
Given: B  5,6,7,8,9,10
C  2,4,6,8,10
D  1,3,5,7,9

A. Find the following:


1. A B 1. _______________________________
2. A'B 2. _______________________________
3.  A  B   B' 3. _______________________________
4. A - C ' 4. _______________________________
5.  A  C ' 5. _______________________________
6. A D 6. _______________________________
7. B  C   A 7. _______________________________
8.  A  D'' 8. _______________________________
9. AU 9. _______________________________
10. B'C   D 10. _______________________________

B. True or False
1. A  Ø = A 1. ________________
2. A' A  U 2. ________________
3. A  U  A 3. ________________
4. A – B = B' A 4. ________________
5. A  U  U 5. ________________

41
MATHEMATICS IN THE MODERN WORLD

C. RELATIONS
Relations abound in daily life: people are related to each other in
many ways as parents and children, teachers and students, employers and
employees, and many others. In business things that are bought are
related to their cost and the amount paid is related to the number of things
bought. We also look at the relation of the prices as the supply is increased
or reduced. In geometry, we say that the area is also related to the volume.
In physics distance travelled is related to the velocity. In general, we relate
one set of information to another. Thus, any correspondence between the
elements of two sets is a relation.

Mathematically; a relation is a correspondence between two things


or quantities. It is a set of ordered pairs such that the set of all first
coordinates of the ordered pairs is called Domain and the set of all the
second coordinates of the ordered pairs is called Range. A relation maybe
expressed as a statement, by arrow diagram, through table, by an
equation, or graphically.

Example:
R = {(1, 2), (2, 4), (3, 6), (4, 8), (5, 10)}

Types of Relations
1. One – to – One relation
2. One – to – Many relation
3. Many – to – One relation

An Equivalence Relation has the following properties:


i. Reflexive:
ii. Symmetric: If , then y .
iii. Transitive : If &y , then .

D. FUNCTIONS
A function is a very important concept in mathematics. A function
is a relation such that each element of the domain is paired with exactly
one element of the range. An equivalent definition is; a function is a
relation in which no two ordered pairs have the same first coordinate. To
denote this relationship, we use the functional notation:

y = f(x)
where f indicates that a function exists between variables x and y.

42
MATHEMATICS IN THE MODERN WORLD

The notation f : is used to denote a function which means that


f is a function with domain A and range B; f(x) = y means that f transform
x (which must be an element of A) into y ( which must be an element of B).

Evaluating a Function
The functional notation y = f(x) allows us to denote specific values of
a function. To evaluate a function is to substitute the specified values of
the independent variable in the formula and simplify.
Example:
When f(x) = 2x – 3, find f(2)
Solution:
f(2) = 2(2) – 3 = 4 – 3
f(2) = 1
a. If f(x) = 2x2 – 3x + 5, find a. f(4)
b. f(-3)
c. c. f(5)
Solution:
a. f(4) = 2 (4)2 – 3(4) + 5 = 2(16) – 12 + 5 = 32 – 12 + 5 = 25
b. f(-3) = 2 (-3)2 – 3(-3) + 5 = 2(9) + 9 + 5 = 18 + 9 + 5 = 32
c. f(5) = 2 (5)2 – 3(5) + 5 = 2(25) – 15 + 5 = 50 – 15 + 5 = 40

Fundamental Operations on Functions


1. The sum/difference of two functions f and g is the functions defined
by
(f ± g)(x) = f(x) ± g(x)
The resultant function is the algebraic sum of the two functions.

2. The product of two functions f and g is the function defined by


(fg)(x) = f(x)g(x)
The resultant function is equal to the product of the separate
images.

3. The quotient of two functions f and g is defined by the function


( )
( ) (x) = ( )
, g(x) ≠ 0
The resultant function is equal to the quotient of the separate
images.

43
MATHEMATICS IN THE MODERN WORLD

Example 1: If f(x) = 8 - 3x and g(x) = 5-x, find


a. (f+g)(x) b. (f-g)(2) c.(fg)(x) d. ( )(x)
Solutions:
a. (f+g)(x) = f(x) + g(x) = (8 – 3x) + (5 – x) = 8 + 5 – 3x – x = 13 – 4x
b. (f – g)x) = f(x) - g(x) = (8 – 3x) – (5 – x) = 8 – 5 – 3x – (-x) = 3 – 2x
c. (fg)(x) = f(x)g(x) = (8 – 3x) (5 – x) = 40 – 23x + 3x2
d. ( )(x) =

Example 2: If f(x) = 8 - 2x and g(x) = 6 - 2x2, find


a. (f+g)(x) b. (f-g)(x) c.(fg)(x)
d. (f+g)(2) e. (f-g)(2) f.(fg)(2)
Solutions:
a. (f+g)(x) = f(x) + g(x)
= (8 – 2x) + (6 – 2x2)
= 8 – 2x + 6 - 2x2 = 8 + 6 – 2x - 2x2
= 14 – 2x - 2x2
b. (f – g)(x) = f(x) - g(x)
= (8 – 2x) - (6 – 2x2)
= 8 – 2x - 6 + 2x2 = 8 - 6 – 2x + 2x2
= 2 – 2x + 2x2
c. (fg)(x) = f(x)g(x)
= (8 – 2x)(6 – 2x2)
= 8(6) + (-2x)6 + 8(-2x2) + (-2x)(-2x2)
= 48 – 12x - 16x2 + 4x3
d. (f+g)(x) = 14 – 2x - 2x2;
(f+g)(2) = 14 – 2(2) – 2(2)2
= 14 – 4 – 8 = 2
e. (f – g)(x) = 2 – 2x + 2x2;
(f – g)(2) = 2 – 2(2) + 2(2)2
=2–4+8=6
f. (fg)(x) = 48 – 12x - 16x2 + 4x3;
(fg)(2) = 48 – 12(2) - 16(2)2 + 4(2)3
= 48 – 24 – 64+ 32 = -8

Inverse of a Function
The inverse of a function is another function that undoes it, and that
it undoes.
For example, the function that takes a number n to n – 5 is the
inverse of the function that takes n to n + 5.
What is the inverse of y = 2x?

44
MATHEMATICS IN THE MODERN WORLD

Since y is in terms of x, then its inverse should be in terms of y. We


look for x = f(y), that is, x = ½ y.

E. OPERATIONS
A binary operation on a set A is a function that takes pairs of
elements of A and produces elements of A from them.

We use the symbol * to denote arbitrary binary operation on a set A.


Four Properties:
1. Commutative: x* y = y *x
2. Associative: x* (y*z) = (x*y)* z
3. Identity: e*x = x *e
4. Inverse: x*y = y*x = e

Computation is associated with operations. It is the manipulation of


numbers and other symbols to arrive at some new mathematical statement.
These other symbols may be letters used to stand for numbers. For
example, in trying to solve a particular problem, we might let X stand for
any number that would meet the conditions of the problem. There are also
symbols to signify what operations to perform on the number symbols. The
most common ones are +, -, x, and / (there are also others). The operations
+ and - are inverses of each other, just as x and / are: One operation
undoes what the other does. The expression a/b can mean "the
quantity a compared to the quantity b," or "the number you get if you
divide a by b," or "a parts of size 1/b." The parentheses in a(b + c) tell us to
multiply a by the sum of b and c. Mathematicians study systems of
numbers to discover their properties and relationships and to devise rules
for manipulating mathematical symbols in ways that give valid results.

A variable is a symbol that assumes many values used to represent


unknown in a problem. A constant is a quantity that assumes specific
value and whose values remain unchanged ―throughout the problem‖. For
example, , in this expression 5 and 6 are the constants and r is the
variable. There are two kinds of constants, the absolute or numerical
constants and the arbitrary constants. Absolute numerical constants are
those, which have same value in all problems, and arbitrary constants are
those, which have the same value throughout any particular problem but
assumes differently in different problems in terms of their values. For
example, in the formula for the circumference of the circle, , is an
absolute value and r is an arbitrary constant. The letter r is arbitrary
because it is the radius of whatever circle would be given. In the area of

45
MATHEMATICS IN THE MODERN WORLD

square , s is a constant for a particular square, hence an arbitrary


constant. A variable is also called a factor of a term. It is usually the literal
coefficient in every algebraic expression.

An algebraic expression is a collection of algebraic symbols, of


constants, variables or finite number of indicated operations involving
variables and constants. For example, 16, -5y, 2m2, 3d2 - 5b, and n/2 – 4x.

An equation is a statement that two algebraic expressions are equal.


Therefore, an equation always includes the quality symbol, =. Examples of
equations are
2x + 4 = 16, 2y = 16, and 4x + 2y = 25 – y

46
MATHEMATICS IN THE MODERN WORLD

NAME: _____________________________________________ SCORE: _______________


SCHEDULE: _________________________________________ DATE: ________________

Exercise 2.1C

A. For each of the following functions, find


a. a general formula for f + g, f – g, fg and .

b. the value of each combination at x = 3 and x = -2.


c. The domain of each combination by using the definitions.

1. f(x) = 4x – 5 and g(x)= 5 – 2x

2. f(x) = 7 – 3x and g(x) = x2 – 3x

3. f(x) = 9 – x2 and g(x) = √

4. f(x) = and g(x) = x2 + 3x + 2

5. f(x) = 3x – 4 and g(x) = √

47
MATHEMATICS IN THE MODERN WORLD

48
MATHEMATICS IN THE MODERN WORLD

Lesson 2.2. Mathematics as a Language

Mathematics students build a foundation of basic understandings in


number, operation, and quantitative reasoning; patterns, relationships,
and algebraic thinking; geometry and spatial reasoning; measurement; and
probability and statistics. They use numbers in ordering, labelling, and
expressing quantities and relationships to solve problems and translate
informal language into mathematical language and symbols. Students use
objects to create and identify patterns and use those patterns to express
relationships, make predictions, and solve problems as they build an
understanding of number, operation, shape, and space. As they progress,
they move from informal to formal language to describe two- and three-
dimensional geometric figures and likenesses in the physical world. And
begin to associate measurement concepts as they identify and compare
attributes of objects and situations.

They later collect, organize, and display data and use information
from graphs to answer questions, make summary statements, and make
informal predictions based on their experiences. As they widen their scope
in mathematics, algorithms are being utilized for generalizations.
Appropriate language and organizational structures such as tables and
charts to represent and communicate relationships, make predictions, and
solve problems become their basic tools. Students select and use formal
language to describe their reasoning as they identify, compare, classify and
generalize concepts.

Mathematics as one progresses, centers on proof, argumentation,


personal perspective and insights, convincing people and making them
understand. Thus, there is elegance in writing mathematics. It is essential
to it. It is like prose and poetry.

Since writing mathematically is essential then after knowing the


fundamental concepts, one needs to practice writing. To start with, just like
any languages definitions are important.

Mathematics holds on to definitions. The concepts needed in


mathematics are defined properly. Definition in mathematics is a concise
statement. It is concise because it plainly contains the basic properties of
an object or concept which unambiguously identify that object or concept.
Thus, the essential characteristics of a good definition are concise, basic
and unambiguously identified. It is concise and not ramble on with

49
MATHEMATICS IN THE MODERN WORLD

extraneous or unnecessary information. It simply involves basic properties,


ideally those that are simply stated and have immediate intuitive appeal. It
should not involve properties that require extensive derivation or proof, or
those that are hard to work with. In order to be complete, a definition must
describe exactly the thing being defined. Nothing more, and nothing less.

GOOD DEFINITION:
A rectangle is a quadrilateral all four of whose angles are right
angles.

POOR DEFINITION:
A rectangle is a parallelogram in which the diagonals have the same
length and all the angles are right angles. It can be inscribed in a
circle and its area is given by the product of two adjacent sides.
This is not CONCISE. It contains too much information, all of which is
correct but most of which is unnecessary.

POOR DEFINITION:
A rectangle is a parallelogram whose diagonals have equal lengths.
This statement is true and concise, but the defining property is not
BASIC. This would work better as a theorem to be proved than as a
definition. In mathematics, assertions of this kind are regarded as
characterizations rather than as definitions.

BAD DEFINITION:
A rectangle is a quadrilateral with right angles.
This is AMBIGUOUS. With some right angles? With all right angles?
There are lots of quadrilaterals that have some right angles but are
not rectangles.

UNACCEPTABLE DEFINITION:
Rectangle: has right angles
This is unacceptable because mathematics is written as English is
written, in complete, grammatical sentences. Such abbreviations
frequently hide major misunderstandings as will be pointed out below.

Though mathematics uses a lot of symbols and terminologies, it is


not plainly putting them together just like the English language. There may
be sentences that are correct in English language but make no sense in
mathematics. Mathematical symbols are a precise form of shorthand. They
have to have meaning for you. To help with understanding you have

50
MATHEMATICS IN THE MODERN WORLD

context and convention. In the English language, we have nouns (name


given to object of interest) and sentences (those that state a complete
thought. Nouns are names of person, place or thing. There are sentences
that are true, not true or those which are sometimes true sometimes false.
A true sentence in English is ―The word ‗mathematics‘ has 11 letters. A
false sentence is that ―The word ‗mathematics‘ has eight letters.‖ The
sentence ―Mathematics is a difficult subject.‖ is sometimes true, sometimes
false.
In Mathematical language, there is also an expression (name given to
mathematical object of interest) and a mathematical sentence (just like the
English language must state a complete thought). Examples of
mathematical expressions are number, set, matrix, ordered pair, average. A
true mathematical statement is 2 + 3 = 5. An example of a mathematical
sentence that is false is that: 2 + 3 = 4. A mathematical statement that is
sometimes true or sometimes false is: x = 2.

Proper writing of mathematical sentences aids to the proper solving


of problems and proofs of theorems or conjectures.
Expression Mathematical expression
A number increased by 7 x + 7
Thrice a number added to 10 3x + 10
one number is four times the other x , 4x
sum of three consecutive integers x + (x + 1) + (x + 2)
Ten less than four times a certain 4x – 10
number
mother is 6 years more than three If x is son‘s age, then, three times
times older than her son son‘s age is 3x and six more than 3x
is 3x + 6

LOGIC AND REASONING


Mathematics started as a practical technique to immediate problems
in life. This was compiled and has been applicable in governance. In the
hands of the ancient Greeks mathematics becomes a systematic body of
knowledge. Mathematics is established as a deductive science in which the
standard of rigorous demonstration is deductive proof.

Aristotle provided a codification of logic which remains definitive for


two thousand years. The axiomatic method is established and is
systematically applied to the mathematics of the classical period by Euclid,
whose Elements becomes one of the most influential books in history. The
next major advances in logic after Aristotle appear in the nineteenth

51
MATHEMATICS IN THE MODERN WORLD

century, in which Boole introduces the propositional (boolean) logic


and Frege devises the predicate calculus. This provides the technical basis
for the logicization of mathematics and the transition from informal to
formal proof.

There is an abundant list of statements in mathematics. These


statements differ from the statements in communication. Some statements
are best confirmed by experiment and other statements are best confirmed
by argument. If I were to tell you, while sitting in a confined room with no
windows, that it is raining outside right now, then there is no amount of
argument that would be as convincing as stepping outside to see for
yourself. The statement ―it is raining'' is not an analytical statement about
the relationship between concepts. It is but a synthetic proposition about
the world that might or might not be true at any given time. In contrast,
mathematical statements are analytical statements that are better proved
by argument than by experiment. This sets the difference of mathematics
from the other sciences. Reasoning in mathematics is different.

Logic is the science of reasoning, proof, thinking, or inference. Logic


allows us to analyze a piece of deductive reasoning and determine whether
it is correct or not – to determine if the argument is valid or invalid. It is a
tool used in mathematical proofs. The rules of logic specify the meaning of
mathematical statements. For instance, these rules help us understand
and reason with statements. Logic is the basis of all mathematical
reasoning, and of all automated reasoning.

Logic is a tool for working with complicated compound statements. It


includes:
 A language for expressing them.
 A concise notation for writing them.
 A methodology for objectively reasoning about their truth or
falsity.
 It is the foundation for expressing formal proofs in all branches of
mathematics
A mathematical proof is an argument that begins with a set of
postulates or assumptions and proceeds to a conclusion by agreed methods
of argument. A deductive argument is one that, if valid, has a conclusion
that is entailed by its premises. Meaning, the truth of the conclusion is a
logical consequence of the premises—if the premises are true, then the
conclusion must be true.

52
MATHEMATICS IN THE MODERN WORLD

A proposition is a declarative sentence (that is, a sentence that


declares a fact) that is either true or false, but not both. As a declarative
sentence, it expresses a complete thought with a definite meaning.

Examples:
1. Saint Louis University is in Baguio City.
2. Quezon City, is the capital of the Philippines.
3. Benguet is part of the Cordillera Administrative Region.
4. 1 + 1 = 2.
5. 2 + 2 = 3.
Not all sentences are considered propositions.

Examples: Consider the following sentences.


1. What day is today?
2. Read the instructions carefully.
3. 5x + 1 = 2.
4. x + 2y = 3z.
Sentences 1 & 2 are not propositions since they are not declarative
sentences.
Sentences 3 & 4 are not propositions because they are neither true
nor false.
Sentences 3 & 4 can be turned into a proposition if we assign values
to the variables.

Mathematically the propositions are converted symbolically. We use


letters to denote propositional variables (or statement variables), that is,
variables that represent propositions, just as letters are used to denote
numerical variables. The conventional letters used for propositional
variables are P, Q, R, S, . . . . The truth value of a proposition is true,
denoted by T, if it is a true proposition, and the truth value of a proposition
is false, denoted by F, if it is a false proposition.

Many mathematical statements are constructed by combining one or


more propositions. New propositions, called compound propositions, are
formed from existing propositions using logical operators or logical
connectives. Logical Connective is a word or symbol that joins two
sentences or propositions to produce a new one.

The five basic logical connectives are conjunction, disjunction,


implication, bi-conditional, and negation.

53
MATHEMATICS IN THE MODERN WORLD

The table below shows the different logical connectives and the
corresponding key words used and the symbol used.

Name Connective (Key Word) Symbol


Conjunction And
Disjunction Or
Implication or Conditional If … then …
Biconditional … if and only if …
Negation Not ~

Conditional statements play an essential role in mathematical


reasoning. They are often seen in the different theorems of mathematics.
There are different ways of expressing p q. The common ones
encountered in mathematics are: ―p implies q‖, ―if p, then q‖, ―if p, q‖, ―p is
sufficient for q‖, ―q if p‖, ―q when p‖, ―a necessary condition for p is q‖, ―q
unless not p‖, ―p only if q‖, ―a sufficient condition for q is p‖, ―q whenever
p‖, ―q is necessary for p‖ and ―q follows from p‖.
Negation of mathematical statement P is denoted by P, read as
―not P‖.
If P is true, then P is not true.

Examples
1. P: The trainees are sleepy.
P: The trainees are not sleepy.

2. Q: I have a new phone.


Q: I do not have a new phone.

TRUTH VALUES
Summary of truth values of compound statements using logical
connectives

P Q P Q P Q P P
T T T T T T

T F F T F F

F T F T T F

F F F F T T

54
MATHEMATICS IN THE MODERN WORLD

The conditional statement can be transformed to new conditional


statements. In particular, there are three related conditional statements
that occur so often that they are given special names. These are the
converse, the contrapositive and the inverse. The contrapositive always has
the same truth value as the conditional.

Assigning P as the antecedent or hypothesis and Q as the


consequent or conclusion; the conditional and the implications or
transformation of it is as follows:

CONDITIONAL: P Q
CONVERSE: Q P
INVERSE:
CONTRAPOSITIVE:

When two compound propositions always have the same truth value
we call them equivalent, so that a conditional statement and its
contrapositive are equivalent. The converse and the inverse of a conditional
statement are equivalent statements but neither is equivalent to the
original conditional statement. An important thing to remember is that one
of the most common logical errors is to assume that the converse or the
inverse of a conditional statement is equivalent to this conditional
statement.

A deductive argument is one that, if valid, has a conclusion that is


entailed by its premises. Meaning, the truth of the conclusion is a
logical consequence of the premises—if the premises are true, then the
conclusion must be true.

A logical argument has three stages:


 Premises: The claims that are given in support of an argument
-- the building blocks of a logical argument. A premise is a
propositional statement which is either true or false.
 Inference: The logical move from one or more premises to
arrive at its conclusion. All inferences must abide by a rule of
inference for an argument to be valid.
 Conclusion: The premise that is the consequence, or product,
of the above premises + inference.*
A conclusion can then itself become a premise (building block) of a
continued or new argument.

55
MATHEMATICS IN THE MODERN WORLD

Symbolic logic symbolizes arguments for simple, efficient assessment


of validity. As arguments get longer and more complex, symbols are
especially important.

Writing an Argument in Symbolic Form

Write Arguments in Symbolic Form and Valid Arguments.


Given propositions:
I have a college degree (p)
I am lazy (q)
Using the propositions and creating an argument:
If I have a college degree, then I am not lazy.
I don‘t have a college degree.
Therefore, I am lazy.
Symbolic form:
If I have a college degree, then I am not lazy ( p →~ q)
I don‘t have a college degree (~ p)
Therefore, I am lazy q
Hypothesis: ((p →~ q) ∧ ~ p)
Conclusion: q
Argument in symbolic form:
((p →~ q) ∧ ~ p)→ q
To test to see if the argument is valid, we take the argument in
symbolic form and construct a truth table. If the last column in the truth
table results in all true‘s, then the argument is valid.

p q ~p ~q ( p →~ q) (( p →~ q)∧ ~ p) (( p →~ q)∧ ~ p)→ q


T T F F F F T
T F F T T F T
F T T F T T T
F F T T T T F

Therefore, this argument is invalid because the last column has a false
item.
Symbolize the argument, construct a truth table, and determine if the
argument is valid.
It will be sunny or cloudy today.
It isn‘t sunny.
Therefore, it will be cloudy.

56
MATHEMATICS IN THE MODERN WORLD

S = It will be sunny
C = It will be cloudy

It will be sunny or cloudy today. S∨C


It isn‘t sunny. ~S
Therefore, it will be cloudy. C

Hypothesis: (S ∨ C) ∧ ~ S
Conclusion: C
S C ~S S∨C (S ∨ C)∧ ~ S ((S ∨ C)∧ ~ S)→C
T T F T F T
T F F T F T
F T T T T T
F F T F F T

This is a valid argument

Rules of Inference

Addition (Add.) Modus Ponens (M.P.)


p p⊃q
∴pvq p
∴q
Simplification (Simp.) Modus Tollens (M.T.)
p·q p⊃q
∴p ~q
∴ ~p
Conjunction (Conj.) Hypothetical Syllogism (H.S.)
p p⊃q
q q⊃r
∴p·q ∴p⊃r
Constructive Dilemma (C.D.) Disjunctive Syllogism (D.S.)
(p ⊃ q) · (r ⊃ s) pvq
pvr ~p
∴qvs ∴q
Destructive Dilemma (D.D.)
(p ⊃ q) · (r ⊃ s)
~q v ~s
∴ ~p v ~r

57
MATHEMATICS IN THE MODERN WORLD

Illustrative Example:

Show that the premises:


―It is not sunny this afternoon and it is colder than
yesterday,‖
―We will go swimming only if it is sunny,‖
―If we do not go swimming, then we will take a canoe trip,‖
and
―If we take a canoe trip, then we will be home by sunset‖

Lead to the conclusion:


―We will be home by sunset.‖

Let: P be the proposition: ―It is sunny this afternoon.‖


Q be the proposition: it is colder than yesterday.‖
R be the proposition: ―We will go swimming.‖
S be the proposition: ―We will take a canoe trip.‖
T be the proposition: ―We will be home by sunset.‖

Then the premises become ¬P ∧ Q, R → P, ¬R → S, and S → T.


The conclusion is simply T.

Constructing an argument to show that the premises lead to the desired


conclusion:
Step: Reason:
1. ¬p ∧ q Premise
2. ¬p Simplification using (1)
3. r → p Premise
4. ¬r Modus tollens using (2) and (3)
5. ¬r → s Premise
6. s Modus ponens using (4) and (5)
7. s → t Premise
8. t Modus ponens using (6) and (7)

Note: A truth table could be used in the proof of the conclusion, that
is to show that each of the four hypotheses are true, the conclusion
is also true. However, since there are five propositional variables, p,
q, r, s, and t, the truth table would have 25 rows or 32 rows.

When the variables in a propositional function are assigned values,


the resulting statement becomes a proposition with a certain truth value.

58
MATHEMATICS IN THE MODERN WORLD

However, there is another important way, called quantification, to create a


proposition from a propositional function. It expresses the words all, some,
many, none and few are used in quantification.

A quantifier is a constructs that specifies the quantity of specimens


in the domain of discourse that satisfy a formula.

TWO TYPES OF QUANTIFICATION


1. Universal – tells us that a predicate is true for every element under
consideration.
2. Existential – tells us that there is one or more element under
consideration for which the predicate is true.
The universal quantification of P(x) is the statement
―P(x) for all values of x in the domain.‖
The notation ― ( ) denotes the universal quantification of P(x).
Here is called the universal quantifier. We read ― ( ) as ―for all x
P(x)‖ or ―for every x P(x).‖ An element for which P(x) is false is called a
counterexample of ― ( ).
― ( ) is True if P(x) is true for every x in the Domain. It is false if
P(x) is False for at least one x in the Domain.

Example:
1. Let P(x) be the statement ―x + 1 > x.‖ What is the truth value of
the quantification x,P(x), where the domain consists of all real
numbers?
Because P(x) is true for all real numbers x, the quantification
x,P(x) is true.

2. Let Q(x) be the statement ―x < 2 ―. What is the truth value of the
quantification x,Q(x), where the domain consists of all real
numbers?
Q(x) is not true for every real number x. Therefore x,Q(x) is
false.

The existential quantification of P(x) is the proposition


―There exists an element x in the domain such that P(x).‖

We use the notation ― ( ) for the existential quantification of


P(x). Here ― is called the existential quantifier.
― ( ) is True if P(x) is true for at least one x in the domain. It is
false if P(x) is false for every x in the domain.
59
MATHEMATICS IN THE MODERN WORLD

NOTE:
A domain must always be specified when a statement x,P(x) is
used. Furthermore, the meaning of x,P(x) changes when the domain
changes. Without specifying the domain, the statement x,P(x) has no
meaning.

Examples:
1. Let P(x) denote the statement ―x>3.‖ What is the truth value of
the quantification xP(x), where the domain consists of all real
numbers?
Because ―x > 3‖is sometimes true ―for instance, when x = 4‖
the existential quantification of P(x), which is x,P(x), is true.
2. Let Q(x) denote the statement ―x = x +1.‖ What is the truth value
of the quantification x,Q(x), where the domain consists of all real
numbers?
Because Q(x) is false for every real number x, the existential
quantification of Q(x), which is x,Q(x), is false.

The quantifiers and have higher precedence than all logical


operators from propositional calculus. For example, ( ) ∨ ( ) is the
disjunction of ( ) and Q(x). In other words, it ( ( )) ∨
( ) rather than ( ( ) ∨ ( ))

Negating Quantified Expressions:


Consider the negation of the statement; ―Every student in your class
has taken a course in calculus.‖
x,P(x), where P(x)is the statement ―x has taken a course in
calculus‖ and the domain consists of the students in your class. The
negation of this statement is ―It is not the case that every student in your
class has taken a course in calculus.‖ This is equivalent to ―There is a
student in your class who has not taken a course in calculus.‖ And this is
simply the existential quantification of the negation of the original
propositional function, namely, x¬P(x).

What is the Negations of the Statement “There is an Honest


Politician”?
Let H(x) denote ―x is honest.‖ Then the statement ―There is an honest
politician‖ is represented by ( ) where the domain consists of all
politicians. The negation of this statement is ( ) which is equivalent
to ( ). This negation can be expressed as ―Every politician is
dishonest.‖

60
MATHEMATICS IN THE MODERN WORLD

De Morgan’s Laws for Quantifiers


Equivalent When is negation
Negation When False?
Statement True?
( ) ( ) For every x, P(x) is false. There is an x for
which P(x) is
true.
( ) ( ) There is an x for which P(x) is true for
P(x) is false. every x.

Multiple Quantifiers such as are said to be


NESTED QUANTIFIERS. These are propositional functions with
multiple quantifiers involving more than one variable.
**Note that everything within the scope of a quantifier can be thought
of as a propositional function.
For example,
x y(x + y = 0)
is the same thing as xQ(x), where Q(x) is yP(x, y), where P(x, y) is x
+ y = 0.

Illustrative Examples:
A. Translate into English the Statement:
x y((x > 0) ∧ (y < 0) → (xy < 0)),
where the domain for both variables consists of all real
numbers.

Solution:
This statement says that for every real number x and
for every real number y, if x > 0 and y < 0, then xy < 0. That
is, this statement says that for real numbers x and y, if x is
positive and y is negative, then xy is negative. This can be
stated more succinctly as ―The product of a positive real
number and a negative real number is always a negative real
number.‖

B. Translate the Statement:


x(C(x) ∨ y(C(y) ∧ F(x, y)))
into English, where C(x) is ―x has a computer,‖ F(x, y) is
―x and y are friends,‖ and the domain for both x and y
consists of all students in your school.

61
MATHEMATICS IN THE MODERN WORLD

Solution:
The statement says that for every student x in your
school, x has a computer or there is a student y such that y
has a computer and x and y are friends. In other words, every
student in your school has a computer or has a friend who
has a computer.

62
MATHEMATICS IN THE MODERN WORLD

NAME: _____________________________________________ SCORE: _______________


SCHEDULE: _________________________________________ DATE: ________________

Exercise 2.2

A. Determine the truth value of all the statements. Write True or False.

1. 47 + 7 = 54 and 41 is a prime number. 1. ______________


2. If 3 and 7 are even, and circles have 4 vertices. 2. ______________
3. 71 + 12 = 83 and 59 is a prime number. 3. ______________
4. If 12 and 64 are even, and square have 4 right angles. 4. ______________
5. 28 + 15 = 43 and 13 is a prime number. 5. ______________
6. If p is "sheep baaw", q is "cats neigh" and r is "cows moo". Then what is the
truth value of (p V q) -> r ? 6. ______________
7. If p is "triangles have 3 vertices", q is "owls hoot" and r is "343 is the
square of 7" what is the truth value of p -> (q ^ r)? 7. ______________
8. Determine the truth value of: "triangles have 3 vertices and
circles have 360 degrees". 8. ______________
9. If p is "cows hoot" and q is "owls baaw". Then what is
the truth value of If p, then q ? 9. ______________
10. Determine the truth value of the negation of the statement:
"If 3 + 7 = 10, then 1 + 2 = 3." 10. _____________

B. Fill in the blanks.

1. If you get a 100 on your final exam, then your teacher will give you an ―A‖
 The ―If‖ statement or p is ______________________________________________.
 The ―then‖ statement
or q is ______________________________________________.

2. If you live in Baguio City, then you live in Benguet.


 The ―If‖ statement or p is ______________________________________________.
 The ―then‖ statement
or q is ______________________________________________.

3. If a number is divisible by 10, then the number ends in zero.


 The ―If‖ statement or p is ______________________________________________.
 The ―then‖ statement
or q is ______________________________________________.

63
MATHEMATICS IN THE MODERN WORLD

64
MATHEMATICS IN THE MODERN WORLD

Chapter

PROBLEM

3 SOLVING AND
REASONING

Objectives: At the end of the chapter, the students are expected to:
a.) Use different types of reasoning to justify statements and arguments
made about mathematics and mathematical concepts
b.) Differentiate inductive reasoning from deductive reasoning
c.) Solve problems employing Polya‘s four steps
d.) Solve problems involving patterns and recreational problems
e.) Increase awareness on the importance of reasoning and problem
solving
f.) Organize their methods and approaches for proving and solving
problems

Lessons:
3.1 Understanding Mathematical Reasoning
3.2 Inductive and Deductive Reasoning
3.3 Polya‘s Four Steps in Problem Solving
3.4 Mathematical Problems involving Patterns
3.5 Recreational Problems using Mathematics

65
MATHEMATICS IN THE MODERN WORLD

Lesson 3.1 Mathematical Reasoning

Reasoning is an essential skill in Mathematics most especially in


problem-solving; it is also important in students‘ activities. Mathematical
reasoning refers to the ability of a person to analyze problem situations
and construct logical arguments to justify his process or hypothesis, to
create both conceptual foundations and connections, in order for him to be
able to process available information. With the development of
mathematical reasoning, students recognize that mathematics makes sense
and can be understood. They learn how to evaluate situations, select
problem-solving strategies, draw logical conclusions, develop and describe
solutions, and recognize how those solutions can be applied.

Reasoning uses statements that consist of two clauses; the first


clause begins with ―if‖ and the second clause starts with ―then‖. These
clauses are commonly known as hypothesis and conclusion and together
they form a statement are referred to as conditional statement. The
conditional statement is represented by ―If p then q‖ here p represents the
―if‖ clause known as the hypothesis and the q represents the ―then‖ clause
known as the conclusion. The conditional statement is represented by the
symbol p ⇒ q which is read as ―if p then q‖.

Examples:

1. If two angles are complementary, then their sum is 90˚.


Hypothesis: If two angles are complementary
Conclusion: Their sum is 90˚.

2. If the number ends in 0 or 2, it is divisible by 2.


Hypothesis: If the number ends in 0 or 2
Conclusion: It is divisible by 2.

3. If two triangles are congruent, then their corresponding parts are


congruent.
Hypothesis: If two triangles are congruent
Conclusion: Their corresponding parts are congruent

4. All vegetarians eat vegetables.


Hypothesis: If one is a vegetarian
Conclusion: He eats vegetables

66
MATHEMATICS IN THE MODERN WORLD

5. He who is lazy shall not eat.


Hypothesis: If one is lazy
Conclusion: He shall not eat

Take note that a conditional statement may sometimes be true or


false. A conditional statement whose hypothesis is true followed by a false
conclusion is false; otherwise true. To show that a conditional statement is
false, a counterexample is needed.

Further, mathematical reasoning is the process of finding the proof


for a certain mathematical statement by using logic and deductions. There
are various types of proofs such as proof by contradiction and proof by
contrapositive. These proofs will take an assumption and then prove the
statement based on the assumption. (https://math.tutorvista.com/math/mathematical-
reasoning.html)

Proof by Contradiction
Proof by contradiction is a process of mathematical reasoning to
prove a given statement. In this method, to prove p we assume ¬p and
derive a contradiction from that. Then since ¬p implies a contradiction, it
cannot hold true. Hence p must be
true.

In a proof by contradiction, the given steps will be followed.


1. Take a proposition p to be proved.
2. Make an assumption that p is false and ¬p is true.
3. Deduce that if ¬p is true, both q and ¬q for a proposition q is true.
4. Hence, we get a contradiction. State that because of the
contradiction, it can't be the case that the statement is false, so it
must be true.
5. It is proved that the assumption p is false is wrong; thus, p is true.

The most common illustration of proof by contradiction is to prove


that √2 is an irrational number.

Statement: √ is an irrational number.

Proof: Assume that √ is a rational number. Then, it can be expressed as a


fraction.
Let us suppose √ = where a and b are relatively prime.

67
MATHEMATICS IN THE MODERN WORLD

Squaring both sides,


2=
2 b2= a2
since a2 is even a will also be even.
4 b2=2(a1)2
similarly, it can be proven that b is even number.

As both a and b are even numbers then they cannot be relatively


prime.

Illustrative Examples:
1. Prove that the negative of an irrational number is irrational.

Solution:
Statement: The negative of an irrational number x is irrational.
Proof: Suppose that -x is a rational number.

Then, we will have two integers a and b such that,


– x = a/b
Multiplying both sides by -1,
x = −a/b

As a is an integer, -a is also an integer. Hence, −a/b is a rational


number which is a contradiction. Our assumption is proved to be wrong.
It is concluded that negative of an irrational number is irrational.
(https://math.tutorvista.com/math/mathematical-reasoning.html)

2. The sum of two even numbers is always even.

Solution:

Let us negate our original statement: The sum of two even numbers
is not always even.

That would mean that there are two even numbers that will give us
an odd number when we add them. By definition, even numbers are evenly
divisible by 2. So we could write our new supposition as:

2a + 2b = c

Even and odd numbers are always integers (no fractions or


decimals), so we know 2a and 2b are integers, which means a and b are
68
MATHEMATICS IN THE MODERN WORLD

also integers. If we divide an even number like 2a by 2, we'll always get a


integer. We also know c is an odd integer, which means it's not evenly
divisible by 2. Now we can factor out a 2 from the left side:

2(a + b) = c

And then divide by 2:

a+b=

The stuff on the left, a + b, must add up to an integer because the


sum of two integers is always another integer. But we already decided cc
must be an odd number, which means we can't divide it evenly by 2. That
means is not an integer.

If a + b is an integer, but isn't an integer, there's no way our


equation is true. The stuff on the left can't possibly equal the fraction on
the right. So, that is a contradiction!
Conclusion: Since the sum of two even numbers 2a and 2b must
always be an integer that's divisible by 2, this contradicts the supposition
that the sum of two even numbers is not always even. Hence, our original
proposition is true: the sum of two even numbers is always even

Proof by Contrapositive

In this method, to prove p q we prove its contrapositive, ¬q ¬p,


instead.

Illustrative Examples:
1. If two angles are complementary, then their sum is 90˚.
Hypothesis: If the sum of two angles is not 90˚
Conclusion: The angles are not complementary

2. If the number ends in 0 or 2, it is divisible by 2.


Hypothesis: If the number is not divisible by 2
Conclusion: It is does not end in 0 or 2

3. If two triangles are congruent, then their corresponding parts are


congruent.

69
MATHEMATICS IN THE MODERN WORLD

Hypothesis: If the corresponding parts of two triangles are not


congruent
Conclusion: The triangles are not congruent

4. All vegetarians eat vegetables.


Hypothesis: If one does not eat vegetables
Conclusion: He is not a vegetarian

5. He who is lazy shall not eat.


Hypothesis: If one eats
Conclusion: He is not lazy

When we interchange the hypothesis and the conclusion of a conditional


statement p ⇒ q, we form the converse statement q ⇒ .

Illustrative Examples:
1. If two angles are complementary, then their sum is 90˚.
Hypothesis: If the sum of two angles is 90˚
Conclusion: The angles are complementary

2. If the number ends in 0 or 2, it is divisible by 2.


Hypothesis: If the number is divisible by 2
Conclusion: It is ends in 0 or 2

3. If two triangles are congruent, then their corresponding parts are


congruent.
Hypothesis: If the corresponding parts of two triangles are congruent
Conclusion: The triangles are congruent

4. All vegetarians eat vegetables.


Hypothesis: If one eats vegetables
Conclusion: He is a vegetarian

5. He who is lazy shall not eat.


Hypothesis: If one does not eat
Conclusion: He is lazy

If we negate the hypothesis and conclusion of a conditional statement p ⇒


q, then we form the inverse statement not q ⇒ .

70
MATHEMATICS IN THE MODERN WORLD

Illustrative Examples:

1. If two angles are complementary, then their sum is 90˚.


Hypothesis: If two angles are not complementary
Conclusion: Their sum is not 90˚.

2. If the number ends in 0 or 2, it is divisible by 2.


Hypothesis: If the number does not end in 0 or 2
Conclusion: It is not divisible by 2.

3. If two triangles are congruent, then their corresponding parts are


congruent.
Hypothesis: If two triangles are not congruent
Conclusion: Their corresponding parts are not congruent

4. All vegetarians eat vegetables.


Hypothesis: If one is not a vegetarian
Conclusion: He does not eat vegetables

5. He who is lazy shall not eat.


Hypothesis: If one is not lazy
Conclusion: He shall eat

71
MATHEMATICS IN THE MODERN WORLD

72
MATHEMATICS IN THE MODERN WORLD

NAME: _____________________________________________ SCORE: _______________


SCHEDULE: _________________________________________ DATE: ________________

Exercise 3.1

A. In each of the following examples, identify both the Hypothesis and


the Conclusion.
1. If you have a coupon, then you will receive a free drink with your pizza.
 Hypothesis: _______________________________________________________________
 Conclusion: ________________________________________________________________

2. If there is no school, then it is the weekend.


 Hypothesis: _______________________________________________________________
 Conclusion: ________________________________________________________________

Not every cause-effect relationship is presented in If-then or conditional form.


Sometimes, you have to interpret the statement to determine the hypothesis and
conclusion before you can write the statement in conditional form.

3. You are not in school during the summer.


 Hypothesis: _____________________________________________________________
 Conclusion: ______________________________________________________________
 Conditional Statement: ___________________________________________________

4. The Beermen will play in the finals after winning this game.
 Hypothesis: _____________________________________________________________
 Conclusion: ______________________________________________________________
 Conditional Statement: ___________________________________________________

B. Fill in the blanks.


Conditional statement: If it is an acute triangle, then all three interior angles have a
measure less than 90°.
1. The hypothesis or p is: ______________________________________________.
2. The negation of
the hypothesis or p is: _________________________________________________.
3. The conclusion or q is: ______________________________________________.
4. The negation of
the conclusion or q is: __________________________________________________.
5. The Contrapositive statement is: ______________________________________.

73
MATHEMATICS IN THE MODERN WORLD

74
MATHEMATICS IN THE MODERN WORLD

Lesson 3.2 Inductive and Deductive Reasoning

Inductive and deductive reasoning are two fundamental forms of


reasoning for mathematicians. Inductive reasoning involves looking for
patterns and making generalizations. For example, students use this type
of reasoning when they look at many different quadrilaterals, and try to list
the characteristics they have in common. The reasoning process is
enhanced by also considering figures that are not quadrilaterals and
discussing how they are different.

QUADRILATERALS NOT QUADRILATERALS

An example of inductive reasoning is, for example, when you notice


that all the dogs you see around you are black and white so you make the
conclusion that all dogs in the world are black and white. Can you say for
certain that this conclusion is correct? No, because it is based on just a few
observations. However, this is the beginning of forming a correct
conclusion, or a correct proof. What this observation has given you is a
starting hypothesis to test out.

Inductive or deductive reasoning can be used when solving


problems. The type of reasoning that forms a conclusion based on the
examination of specific examples is called inductive reasoning. The
conclusion formed by using inductive reasoning is often called a
conjecture, since it may or may not be correct.

75
MATHEMATICS IN THE MODERN WORLD

Illustrative Examples:

1. Consider the following procedure:


a. Pick a number.
b. Multiply the number by 8,
c. Add 6 to the product
d. Divide the sum by 2, and
e. Subtract 3.

Complete the above procedure for several different numbers. Use


inductive reasoning to make a conjecture about the relationship between
the size of the resulting number and the size of the original number.

2. Consider the following procedure:


a. Pick a number.
b. Multiply the number by 9,
c. Add 15 to the product,
d. Divide the sum by 3, and
e. Subtract 5.

Complete the above procedure for several different numbers. Use


inductive reasoning to make a conjecture about the relationship between
the size of the resulting number and the size of the original number.

3. Consider the following procedure:


a. List 1 as the first odd number
b. Add the next odd number to 1.
c. Add the next odd number to the sum.
d. Repeat adding the next odd number to the previous sum.

Another type of reasoning is called deductive reasoning. Deductive


reasoning is distinguished from inductive reasoning in that it is the process
of reaching a conclusion by applying general principles and procedures.
Deductive reasoning involves making a logical argument, drawing
conclusions, and applying generalizations to specific situations. For
example, once students have developed an understanding of "triangle," they
apply that generalization to new figures to decide whether or not each is a
triangle. The conclusions reached by this type of reasoning are valid and
can be relied on.

76
MATHEMATICS IN THE MODERN WORLD

Illustrative Examples:
1. If a number is divisible by 2, then it must be even.
12 is divisible by 2.
Therefore, 12 is an even number.

2. All Mathematics teachers know how to play Sudoku.


Resty is a Math teacher.
Therefore, Resty knows how to play Sudoku.

3. If a student is a DOST scholar, he receives a monthly allowance.


If a student receives a monthly allowance, his parents will be
happy.
Therefore, if a student is a DOST scholar, his parents will be
happy.

4. If ∠A and ∠B are supplementary angles, their sum is 180º.


If m∠A = 100º, then m∠B = 80º

Logic Puzzles can be solved by deductive reasoning. A chart that enables


us to display the given information in a visual manner helps in the
solution.

Illustrative Examples:
1. Each of four neighbors, Mark, Zen, Linda, and Roy, has a different
occupation (teacher, banker, chef, or broker).

From the following clues, determine the occupation of each neighbor.

CLUES
1. Zen gets home from work after the banker but before the broker.
2. Linda, who is the last to get home from work, is not the teacher.
3. The dentist and Linda leave for work at the same time.
4. The banker lives next door to Roy.

Solution:
From clue 1, Zen is neither the banker nor the broker.
From clue 2, Linda is not the teacher.

77
MATHEMATICS IN THE MODERN WORLD

We know from clue 1 that the banker is not the last to get home, and
we know from clue 2 that Linda is the last to get home; therefore, Linda is
not the banker
From clue 3, Linda is not the broker.
As a result, Linda is the Chef.
Since Linda is the Chef, it could not be Zen.
Zen, therefore, is the Teacher.
From clue 4, Roy is not the banker.
And since Linda is the Chef and Zen is the Teacher, Roy must be the
Broker.
Mark is the Banker, the only occupation not filled up.

Teacher Banker Chef Broker

Mark x / x x

Zen / x x x

Linda x x / x

Roy x x x /

2. Daisy, Karen, James, and Rudy were recently elected as the new class
officers (president, vice president, secretary, treasurer) of the sophomore
class at Sunbeam College.
From the following clues, determine which position each holds:
1. Rudy is younger than the president but older than the treasurer.
2. Daisy and the secretary are both the same age, and they are the
youngest members of the group.
3. James and the secretary are next-door neighbors.

Vice
President President Secretary Treasurer
Daisy x x x /
Karen x x / x
James / x x x
Rudy x / x x

78
MATHEMATICS IN THE MODERN WORLD

Answer:
President -- James
Vice –President –Rudy
Secretary - Karen
Treasurer – Daisy

Einstein Puzzle:
Watch ―Can you solve Einstein’s Riddle – Dan Van der Vieren.mp4”

Einstein Puzzle Clues


1. The Brit lives in the house with the red walls.
2. The Swede has a dog.
3. The Dane drinks tea.
4. The house with green walls is directly to the left of the house with
white walls.
5. The owner of the house with green walls drinks coffee.
6. The person who smokes Pall Mall cigars owns a bird.
7. The owner of the house with yellow walls smokes Dunhill.
8. The man living in the center house drinks milk.
9. The Norwegian lives in the first house.
10. The man who smokes Blends lives next to the cat owner.
11. The horse‘s owner lives next to the man who smokes Dunhill.
12. The man who smokes Blue Master drinks root beer.
13. The German smokes Prince.
14. The Norwegian lives next to the house with blue walls.
15. The man who smokes Blends has a next-door neighbor who
drinks water.

Who stole the fish?

House
Owner Animal Drink Cigar Wall color
Number
1 Norwegian cat water Dunhill yellow
2 Dane horse tea Blends blue

3 Brit bird milk Pal Mall


red
4 German coffee Prince green
5 Swede dog Root beer Blue Master White

79
MATHEMATICS IN THE MODERN WORLD

Answer: The German got the fish!

80
MATHEMATICS IN THE MODERN WORLD

NAME: _____________________________________________ SCORE: _______________


SCHEDULE: _________________________________________ DATE: ________________

Exercise 3.2
A. Classify the reasoning employed in the following arguments as INDUCTIVE
or DEDUCTIVE.

1. All even numbers are divisible by 2. Twenty-eight is even. Therefore, 28 is


divisible by 2.
(1) ______________

2. 3,6,9,12,15,____. The next term is going to be 18.


(2) ______________

3. Since all squares are rectangles, and all rectangles have four sides, all
squares have four sides.
3. _______________

4. For any right triangle, the Pythagorean Theorem holds. 2) ABC is a right
triangle, therefore for ABC the Pythagorean Theorem holds.
4. _______________

5. The population of Baguio City has risen steadily for the past 40 years.
It is logical to predict that the population of Baguio City will also rise next year.
5. _______________

6. Two figures are said to be congruent if they have the same shape and size or
if one has the same shape and size as the mirror image of the other. My figure
is the mirror image of my mirror image, therefore my figure and my mirror
image are congruent.
6. _______________

7. If x = 4
And if y = 1
Then 2x + y = 9
7. ___________________

8. Based on a survey of 3300 randomly selected registered voters, 56.2%


indicate that they will vote for the incumbent officials in the upcoming election.
Therefore, approximately 56% of the votes in the upcoming election will be for
the incumbent.
8. ___________________

81
MATHEMATICS IN THE MODERN WORLD

9. Jack is taller than Jill. Jill is taller than Joey. Therefore, Jack is taller than
Joey.
9. _________________

10. It usually takes 2–3 days for a delivery to ship from the warehouse to your
door via most major shipping services. You ordered on Tuesday morning, so it‘s
safe to assume your package will arrive Thursday or Friday.
10. ______________________

82
MATHEMATICS IN THE MODERN WORLD

Tower of Hanoi
Inductive and Deductive Reasoning: The Tower of Hanoi

The Tower of Hanoi is ―a logical puzzle, frequently studied in


cognitive psychology and used as a test of problem-solving ability,
consisting of three pegs, on one of which are placed a number of discs of
varying diameter, the largest at the bottom and the smallest at the top
(Oxford Reference).

The puzzle was popularized by French mathematician Edouard


Lucas (184291) and marketed as a toy in 1883. A legend that comes with it
says, ―There is a temple in the vicinity of the Kashi Vishwanath shrine
(famous pilgrimage center of Lord Shiva and the river Goddess Gang) in
Varanasi, India, where there are three long-time worn posts located in a
large room inside the temple. The posts are surrounded by 64 golden disks,
which have to be moved by Brahmin priests as per the command of an
ancient prophecy serving the incontrovertible rules of Lord Brahma. The
legend further says that when the last move of this puzzle would be
completed, the world shall come to an end.‖ (From, De Parville, La Nature,
1884, part I, pp.285-286; as translated in W. W. Rouse Ball and H. S. M.
Coxeter, Mathematical Recreations and Essays, 13th edition, Dover, NY,
1987, p. 317.)

―We will say immediately that it would be necessary to perform


successively a number of moves equal to 18 446 744 073 709 551 615
which would require more than five billion centuries!‖(Lucas, as cited in
Stockmeyer, 1998).

EXPLORATION

A Famous Puzzle, The Tower of Hanoi is a puzzle invented by


Edouard Lucas in 1883. The puzzle consists of three pegs and a number of
disks of distinct diameters stacked on one of the pegs such that the largest
disk is on the bottom, the next largest is placed on the largest disk, and so
on as shown in the next figure. The object of the puzzle is to transfer the
tower to one of the other pegs. The rules require that only one disk be
moved at a time and that a larger disk may not be placed on a smaller disk.
All pegs may be used. Determine the minimum number of moves required to
transfer all of the disks to another peg for each of the following situations.

83
MATHEMATICS IN THE MODERN WORLD

a. You start with only one disk.


b. You start with two disks.
c. You start with three disks. (Note: You can use a stack of various size
coins to simulate the puzzle, or you can use one of the many websites
that provide a simulation of the puzzle.)
d. You start with four disks.
e. You start with five disks.
f. You start with n disks.

Complete the table below based from the result of the activity.
NUMBER OF CUMULATIVE SUM OF
NUMBER OF DISK(S)
MOVES MOVES
1

3
4
5

7
8

Write down any observation or conjecture from the activity.


__________________________________________________________________________
__________________________________________________________________________
__________________________________________________________________________
__________________________________________________________________________
__________________________________________________________________________

84
MATHEMATICS IN THE MODERN WORLD

Lesson 3.3 Polya’s Problem Solving

The National Council of Teachers of Mathematics (NCTM) posits that


problem solving plays an important role in mathematics and should have a
prominent role in the mathematics education of K-12 students. The term
"problem solving" refers to mathematical tasks that have the potential to
provide intellectual challenges for enhancing students' mathematical
understanding and development. NCTM pointed out that people who can
reason and think analytically tend to: (a) note patterns, structure, or
regularities in both real-world situations and symbolic objects; (b) ask if
those patterns are accidental or if they occur for a reason; (c) conjecture
and prove

Problem-solving is an important skill not only in dealing with


Mathematics, but also in making decisions in life. Decision-making is a
significant part of problem-solving. Students should be taught how to solve
problems logically and accurately; and in the process, they discover that
there are various ways to solve a problem.

George Polya, a Hungarian mathematician, described the methods of


problem solving in his book How to Solve It (1945). This systematic
process for solving problems is now referred to as the Polya 4-Step Problem-
Solving Process. This process is discussed in this lesson to help students
develop an action plan for addressing problems.

Polya‘s four steps in Problem-Solving are as follows:


1. Understanding the Problem.
Before anything else, understand the problem - have a full grasp of
what are known and not known. To help you understand the
problem, consider the following:
(a) restate the problem in your own words
(b) determine exactly what is asked for
(c) identify or list the given data, conditions, and information
(d) identify the unknown(s)

2. Devise a Plan – strategize.


Pólya mentions that there are many reasonable ways to solve
problems. The skill at choosing an appropriate strategy is best
learned by solving various problems. Applying strategies to devise a
plan requires skill and own judgment. Some strategies are as
follows:

85
MATHEMATICS IN THE MODERN WORLD

(a) Make an organized list, a table, a chart, or a diagram


illustrating the given and the unknown parts
(b) Formulate an equation or a model showing the relationship
between the given data and the unknown
(c) Look for a pattern; guess and check
(d) Examine related problems and determine if the same
technique can be applied
(e) Work backwards

3. Carry out the plan


After devising a plan, the next logical step is to carry out that plan.
(a) Implement the strategy in Step 2 and perform any necessary
actions or computations.
(b) Check each step of the plan as you proceed; this may be
intuitive checking or a formal proof of each step.
(c) Keep an accurate record of your steps as you implement your
devised plan.
(d) Persist with the plan that you have chosen, and if it continues
not to work, discard it and choose another.

4. Look back
Pólya mentions that much can be gained by taking the time to
reflect, examine, and look back at what you have done - what
worked and what didn't; doing this will enable you to predict what
strategy to use to solve future problems, if these relate to the original
problem.

Ensure that the solution is consistent with the facts of the problem.
 Interpret the solution in the context of the problem.
 Ask yourself whether there are generalizations of the solution
that could apply to other problems.
(a) Examine the solution obtained. Check the results in the
original problem (in some cases, this will require a proof).
(b) Interpret the solution in terms of the original problem. Find
out if your answer makes sense or is reasonable. Ensure that
the solution is consistent with the facts of the problem.
(c) Determine whether there is another method of finding the
solution.
(d) If possible, determine other related or more general problems
for which the techniques will work; find out if there are

86
MATHEMATICS IN THE MODERN WORLD

generalizations of the solution that could apply to other


problems

Illustrative Examples:

1. One number is 7 more than another. Twice the larger is equal to four
times the smaller decreased by 2. Find the numbers.

Step 1: Understand the Problem


We are looking for two numbers wherein one is 7 more than another,
and twice the larger is equal to four times the smaller decreased by 2.

Step 2: Devise a plan

We can use the ―Formulate an equation‖ strategy


x = smaller number
x + 7 = larger number

Equation: 2(x + 7) = 4x – 2

Step 3: Carry out the plan

2(x + 7) = 4x – 2

2x + 14 = 4x – 2

Subtract 4x and subtract 14 from the both sides of the equation

2x – 4x + 14 – 14 = 4x – 4x – 2 – 14

– 2x = – 16 then divide both sides by – 2

x = 8 and x + 7 = 8 + 7 = 15

Step 4: Look back

15 is 7 more than 8; twice 15, which is 30, is four times 8 less 2.


Thus, the final answer is: The smaller number is 8 and the larger
number is 15.

87
MATHEMATICS IN THE MODERN WORLD

2. Anne is 2 years older than Betty. Last year Anne was 2 times as old as
Betty. How old is Anne?

Step 1: Understand the Problem


We are looking for the age of Anne at present. She is now 2 years
older than Betty, and last year she was twice as old as Betty.

Step 2: Devise a plan


x = age of Betty now x – 1 = age of Betty last year
x + 2 = age of Anne now x + 2 – 1 = x + 1 = age of Betty last year

We can also use a table such as this:

Age now Age last year


Betty x x-1
Anne x+2 x+1

Last year, Anne was twice as old as Betty


Thus, the equation is: x – 1 = 2(x + 1)

Step 3: Carry out the plan


x – 1 = 2(x + 1)
x – 1 = 2x + 2
x = 3 and x + 2 = 5

Step 4: Look back


If Anne is 5 years old now, Betty is 3 years old. Last year, Anne was
4 and Betty was 2, that is, Anne was twice as old as Betty.

Final answer: Anne is 5 years old at present.

3. Jerry is 7 years older than Jan. In three years Jerry will be twice as old
as Jan. Find their present ages.

Step 1: Understand the Problem


We are looking for the present ages of Jerry and Jan. At present,
Jerry is 7 years older than Jan. Three years from now, Jerry will be twice
as old as Jan.

88
MATHEMATICS IN THE MODERN WORLD

Step 2: Devise a plan


Age at present Age 3 years from now
Jan x x+3
Jerry x+7 x + 10

Three years from now, Jerry will be twice as old as Jan


Thus, the equation is: x + 10 = 2(x + 3)
x + 10 = 2x + 6
x = 4 and x + 7 = 11

Step 3: Carry out the plan


x + 10 = 2(x + 3)
x + 10 = 2x + 6
x = 4 and x + 7 = 11

Step 4: Look back


11 (Jerry‘s age) is 7 more than 4 (Jan‘s age); in three years, Jan will
be 7 while Jerry will be 14 which is twice the age of Jan.

Thus, the final answer is: The present age of Jan is 4 and Jerry‘s age is 11.

4. Tonio and Mao left at 8A.M. from the same point, Tonio traveling east at
an average speed of 50 mph and the Mao travelling south at an average
speed at 60 mph. At what time to the nearest minute will they be 300 miles
apart?

Step 1: Understand the Problem


Tonio and Mao start at the same time, from the same point in
different directions (Tonio to the east and Mao to the south) and drove at
different average speeds. We are looking for the time spent after they are
300 miles apart, assuming that they drove continuously.

Step 2: Devise a plan


Make a diagram showing their directions. Recall that distance equals
the product of speed/rate and time, that is, d = rate x time. The two cars
are traveling in directions that are at right angles, so the Pythagorean
Theorem holds: the square of the hypotenuse is equal to the sum of the
squares of the two legs of the triangle. Letting x and y be the distances
traveled by the two cars in t hours.

89
MATHEMATICS IN THE MODERN WORLD

EAST
After t hours, the distances x and
y, in miles per hour, traveled by
y the two cars are given by:
D
Tonio: x = 50t
Mao: y = 60t

SOUTH Using Pythagoras‘ theorem, D2 = x2 + y2

Step 3: Carry out the plan


D2 = x 2 + y2
D =√( ) ( )
300 = √
300 = √ 300 = 78.102t
t = 3.8411 hours or 3 hours and 51 minutes (to the nearest minute)

Step 4: Look back


3002 = 2500(3.8411)2 + 3600 (3.8411)2
90,000 ≈ 90,000 (rounded to the nearest integer)

Final answer: Tonio and Mao will be 300 miles apart at 11:51 A.M.

5. Here are the first five terms of a number sequence: 2, 7, 12, 17, 22.
a) Write down the next term in the sequence.
b) 45 is not a term in this number sequence. Explain why

Step 1: Understand the Problem


We are looking for a pattern in the given five numbers, and then
determine the next number.

Step 2: Devise a plan


We use the strategy ―Look for a pattern; guess and check‖. Since 7–
2 = 5 and also 12 – 7 = 5, then maybe 5 should be continuously added
to get the next number in the sequence.

90
MATHEMATICS IN THE MODERN WORLD

Step 3: Carry out the plan


Check if the guess (adding 5 to get the next number) is correct.
2+5=7
7 + 5 = 12
12 + 5 = 17
17 + 5 = 22
22 + 5 = 27

So, the next term or the 6th term in the sequence is 27.

Step 4: Look back


7–2=5
12 – 7 = 5
17 – 12 = 5
22 – 17 = 5
27 – 22 = 5

Final answer: the next term in the sequence is 27


Looking at the pattern, adding the number 5 five times to the first
number in the sequence which is 2, we get 27. That is, 2 + 5(5) = 27. Note
that 5 is added five times to get the 6th term. Looking back, add 5 four times
to get the 5th term, that is, adding the 5 to the first term (n – 1) times.

Generalization:

We can make a generalization to find the nth term in the sequence:


Let a = the first number in the sequence;
n = the number of terms;
an = the nth term;
d = common difference (the term which is added to the preceding term
to get the next term)

So, if 27 = 2 + 5(5), then an = a + d(n – 1)


To answer the second part ―45 is not a term in this number
sequence‖, use the equation an = a + d(n – 1)
If 45 is a term in the sequence, which term is it?

45 = 2 + 5n – 5 48 = 5n n = 9.6
Since n is not a counting number, then 45 is not a term in the sequence.

91
MATHEMATICS IN THE MODERN WORLD

Lesson 3.4 Mathematical Problems involving Patterns

An ordered list of numbers such as

– 3, 1, 5, 9, 13 . . .

Is called a sequence. The numbers in a sequence that are separated


by commas are the terms of the sequence. The above sequence is called an
arithmetic sequence. Sometimes the words ‗series’ or ‗progression’ are
used in place of sequence. In the above sequence, – 3 is the first term
represented by a1, 1 is the second term represented by a2, 5 is the third
term represented by a3, 9 is the fourth term, and 13 is the fifth term. The
three dots ―...‖ indicate that the sequence continues beyond 13, which is
the last written term. It is customary to use the subscript notation an to
designate the nth term of a sequence. In example 5 above, it was shown
that an = a + d( n – 1).

Illustrative Examples:

1. Mic decided to save money for one week from his allowance. Each day
he saves 12 pesos more than the previous day. If he started saving 8
pesos in the first day, how much will he set aside in the 5 th day? After a
week, how much would he have saved?

Solution:
The sequence starts with 8 then add 12 until the fifth day. Thus, the
sequence consists of:
8, 20, 32, 44, 56

Therefore, on the fifth day, Mic will set aside 56 pesos. After a week
(seven days), he has saved 308 pesos.

3. Find the 7th term in the sequence 5, 14, 27, 44, 65...

14 – 5 = 9
27 – 14 = 13
44 – 27 = 17
65 – 44 = 21

92
MATHEMATICS IN THE MODERN WORLD

Note that the pattern is adding 4 more than the number added in
the previous number. So the next to be added is 25, and then 29

Thus, 65 + 25 = 90 Then, 90 + 29 = 119

Answer: The 7th term is 119.

An ordered list of numbers such as


3, -3/2, 3/4, -3/8, 3/16...
Is called a geometric sequence. Sequences of numbers that follow a
pattern of multiplying a fixed number, r, from one term to the next are
called geometric sequences. The following sequences are geometric
sequences:
Sequence A: 1 , 2 , 4 , 8 , 16 , ... r=2
Sequence B: 0.01 , 0.06 , 0.36 , 2.16 , 12.96 , ... r=6
Sequence C: 16 , -8 , 4 , -2 , 1 , ... r = – 1/2

The r-value can be calculated by dividing any two consecutive terms


in a geometric sequence. The formula for calculating r is...

...where n is any positive integer greater than 1.

In the above examples,


n Sequence A Sequence B Sequence C
21 - 1
= 20 = 1 (0.01)61 - 1 = (0.01)60 = 16)(-1/2)1 - 1 = (16)(-1/2)0 =
1
(0.01)(1) = 0.01 (16)(1) = 16
22 - 1 = 21 = 2 (0.01)62 - 1 = (0.01)61 = (16)(-1/2)2 - 1 = (16)(-1/2)1 =
2
(0.01)(6) = 0.06 (16)(-1/2) = -8
23 - 1 = 22 = 4 (0.01)63 - 1 = (0.01)62 = (16)(-1/2)3 - 1 = (16)(-1/2)2 =
3
(0.01)(36) = 0.36 (16)(1/4) = 4
24 - 1 = 23 = 8 (0.01)64 - 1 = (0.01)63 = (16)(-1/2)4 - 1 = (16)(-1/2)3 =
4
(0.01)(216) = 2.16 (16)(-1/8) = -2
25 - 1 = 24 = 16 (0.01)65 - 1 = (0.01)64 = (16)(-1/2)5 - 1 = (16)(-1/2)4 =
5
(0.01)(1296) = 12.96 (16)(1/16) = 1
nth an = 2n - 1 an = (0.01)6n - 1 n = (16)(-1/2)
n-1

term

In general, to find the nth term of a geometric sequence, the formula


an = a1n – 1 is used.

93
MATHEMATICS IN THE MODERN WORLD

Lesson 3.5 Recreational Problems using Mathematics

Sudoku
The popular Japanese puzzle game Sudoku is based on the logical
placement of numbers. A Sudoku puzzle is defined as a logic-based,
number-placement puzzle. The objective is to fill a 9×9 grid with digits in
such a way that each column, each row, and each of the nine 3×3 grids
that make up the larger 9×9 grid contains all of the digits from 1 to 9. Each
Sudoku puzzle begins with some cells filled in. The player uses these seed
numbers as a launching point toward finding the unique solution.

It is important to stress the fact that no number from 1 to 9 can be


repeated in any row or column (although, the can be repeated along the
diagonals).

Sudoku tips that you can use to improve your Sudoku skills:
 Tip 1: Look for rows, columns of 3×3 sections that contain 5 or more
numbers. Work through the remaining empty cells, trying the
numbers that have not been used. In many cases, you will find
numbers that can only be placed in one position considering the
other numbers that are already in its row, column, and 3×3 grid.
 Tip 2: Break the grid up visually into 3 columns and 3 rows. Each
large column will have 3, 3×3 grids and each row will have 3, 3×3
grids. Now, look for columns or grids that have 2 of the same
number. Logically, there must be a 3rd copy of the same number in
the only remaining 9-cell section. Look at each of the remaining 9
positions and see if you can find the location of the missing number.

To try playing Sudoku, visit https://www.funbrain.com/games/sudoku


https://www.miniclip.com/games/sudoku/en/

2 9 8 5
4 7 1 3
3 9 6 4 7
2 5 6 4
8 4 3 2 1
9 7 1 8 6
6 7 5 1 3
9 1 4 5
2 3 6 8

94
MATHEMATICS IN THE MODERN WORLD

Solution:
1 7 2 9 8 3 5 6 4

4 6 8 5 7 2 9 1 3

5 3 9 6 1 4 8 7 2

2 1 3 8 5 6 4 9 7

8 4 6 3 9 7 2 5 1

9 5 7 2 4 1 3 8 6

6 8 4 7 2 5 1 3 9

3 9 1 4 6 8 7 2 5

7 2 5 1 3 9 6 4 8

Magic Squares
A magic square of order n is an arrangement of numbers in a square
such that the sum of the n numbers in each row, column, and diagonal is
the same number. (from TIMSS 2011)

Pat has red tiles and black tiles. Pat uses the tiles to make square
shapes.

The 3 x 3 shape has 1 black tile The 4 x 4 shape has 4 black


and 8 red tiles. tiles and 12 red tiles

R R R R R R R

R B R R B B R

R R R R B B R

- Black tile R R R R
B

R - Red tile

95
MATHEMATICS IN THE MODERN WORLD

KenKen Puzzles

KenKen is an arithmetic-based logic puzzle that was invented by the


Japanese mathematics teacher Tetsuya Miyamoto in 2004. The noun ―ken‖
has ―knowledge‖ and ―awareness‖ as synonyms. Hence, KenKen translates
as knowledge squared, or awareness squared.

KenKen puzzles are similar to Sudoku puzzles, but they also require
you to perform arithmetic to solve the puzzle.

Rules for solving a Ken Ken puzzle:

For a 3 x 3 puzzle, fill each box (square) of the grid with one of the numbers
1, 2, or 3.

For a 4 x 4 puzzle, fill each box (square) of the grid with one of the numbers
1, 2, 3, or 4.

For an xn puzzle, fill each box (square) of the grid with one of the numbers
1, 2, 3… n.

 Do not repeat a number in any row or column.


 The numbers in each heavily outlined set of squares, called cages,
must combine (in some order) to produce the target number in the
top lest corner of the cage using the mathematical operation
indicated.
 Cages with just one square should be filled in with the target
number.
 A number can be repeated within a cage as long as it is not in the
same row or column.

96
MATHEMATICS IN THE MODERN WORLD

Example: A 4 x 4 Ken Ken puzzle with 8 cages

Solution

6x 7+ 6x 7+

2 1 3 4
2 8x 2 8x

3 2 4 1
4x 12x -1
4x 12x -1
1 4 2 3
1
1
4 3 1 2
Other examples:

16x 1-

1 4 2 3

2 4+

4 2 3 1

18x 1 2÷

2 3 1 4

3-

3 1 4 2

97
MATHEMATICS IN THE MODERN WORLD

11+ 2÷ 20x 6x

5 6 3 4 1 2

3- 3÷

6 1 4 5 2 3

240x 6x

4 5 2 3 6 1

6x 7+ 30x

3 4 1 2 5 6

6x 9+

2 3 6 1 4 5

8+ 2÷

1 2 5 6 3 4

98

Potrebbero piacerti anche